Missed MBE questions

Lakukan tugas rumah & ujian kamu dengan baik sekarang menggunakan Quizwiz!

A defendant was involved in an accident in which her car struck the rear end of the car driven by the plaintiff. The police issued tickets to the defendant, charging her with reckless driving and speeding. When the defendant's case came before the traffic court, her attorney entered into a plea bargain with the prosecutor. Under the plea bargain, the defendant agreed to plead guilty to speeding and to pay a fine of $100, and the prosecution agreed to drop the reckless driving charge. Accordingly, the defendant pleaded guilty and the court fined her $100. In the later civil suit, where the plaintiff is seeking damages from the defendant for personal injuries, is the guilty plea before the traffic court admissible? A Yes, because it is a statement by an opposing party. B Yes, because it is a statement against interest. C No, because there is a public policy in favor of plea bargaining to promote court efficiency. D No, because no felony was involved.

A

A landowner included in his will a provision giving "all of my property, both real and personal, wherever situated, to my widow for life, and after her death to any of our children who may survive her." What is the gift to the children? A A contingent remainder. B A vested remainder. C A shifting executory interest. D Void, as violating the Rule Against Perpetuities.

A

A man who belonged to an ancient religion whose rituals require the use of bald eagle feathers traveled to an area where bald eagles were known to roost. After searching the area, he found a fallen eagle feather and returned home. A few weeks later, the man showed the feather to an acquaintance, who happened to be a state park ranger, and explained how the feather was obtained. The ranger informed the man that a state anti-poaching law makes any possession of a bald eagle feather without a special permit a crime. The ranger then cited the man for possession of the feather and confiscated it. At the man's trial for violating the state bald eagle feather possession statute, which of the following constitutional arguments is most appropriate for the prosecution to make? A The statute is a neutral law that only incidentally burdens the man's rights under the First Amendment. B The Free Exercise Clause applies only to belief and not to conduct. C The government has a substantial and important interest in protecting bald eagles and there is no other feasible way to achieve the legislative purpose. D Making an exception for the man on religious grounds would violate the Establishment Clause of the First Amendment.

A

A plumber working for a company providing plumbing services to commercial and industrial establishments was required to be "on call" for emergency plumbing services 24 hours a day, and was required to drive his company van home each night so he would have all of his tools and equipment at hand for any calls. However, he was not permitted to use the company van for personal errands. On his way home one afternoon, he took a detour toward a supermarket a few blocks away to pick up some items for dinner. While entering the supermarket parking lot, he drove negligently and struck a pedestrian, seriously injuring him. The pedestrian filed suit against the plumber's company in a jurisdiction that maintains traditional common law rules regarding contribution and indemnity, and the jury awarded him $100,000 in damages, which the company paid. If the company sues the plumber to recoup its loss in the lawsuit, which party will prevail? A The company can recover 100% of the judgment as an indemnity, because the plumber was negligent, not the company. B The company will prevail, because the company had a rule against using company vehicles for personal errands. C The company will not prevail, because the company has already been found liable under principles of vicarious liability in the lawsuit by the pedestrian. D The company will not prevail, because the company required the plumber to be "on call" 24 hours a day.

A

To help alleviate discrimination in private contracts, Congress passed a bill providing: "It shall be unlawful to discriminate against minority race members in the making and enforcement of any public or private contract, of every kind whatsoever. Any person whose rights under this statute are violated may bring a cause of action against the party that has so violated the person's rights in the federal district court for the district in which he resides, seeking treble damages or $1,000, whichever is greater." Several large banks that have been accused of discriminatory loan practices challenge the federal statute. If the court finds that Congress had the power to enact the statute, the court most likely will find that the power arose from which of the following? A The Thirteenth Amendment. B The Contract Clause. C The Fourteenth Amendment. D The Commerce Clause.

A

A former shareholder of a corporation filed an action against the corporation's board of directors, alleging that the directors' wrongful actions diminished the value of the corporation and its stock. The former shareholder served a request for production of documents that included a request for any documents relating to the value of the corporation. The corporation produced a number of documents, but withheld certain documents that were written by its attorneys, claiming that the documents were protected from discovery under the attorney-client privilege. Without contacting or conferring with the corporation or its lawyer, the former shareholder filed a motion to compel production of the withheld documents, claiming that they were not covered by the privilege. The court ruled that the documents were not privileged and had to be produced. What will be the likely result if the former shareholder seeks to recover costs or attorneys' fees related to the motion or to have sanctions imposed on the corporation? A The shareholder may not recover costs or attorneys' fees or have sanctions imposed. B The shareholder may recover the costs of asserting the motion to compel but may not recover any attorneys' fees or have sanctions imposed. C The shareholder may recover the costs and attorneys' fees incurred in asserting the motion to compel, but may not have sanctions imposed. D The shareholder may recover costs and attorneys' fees incurred in asserting the motion to compel, and may have sanctions imposed.

A If a party fails to provide discovery or provides incomplete discovery, including disclosures and answers to interrogatories and deposition questions, the other party may move to compel discovery. However, a motion to compel must certify that the moving party has made a good faith attempt to confer with the opponent to obtain the discovery without court intervention. The certification (and an actual attempt at resolving the discovery dispute without court intervention) is a prerequisite to an award of reasonable expenses (which includes attorneys' fees). Here, the movant cannot certify in a motion to compel that he made a good faith effort to obtain discovery without court action because the facts indicate that he made no such effort. Sanctions generally may be imposed only for violating a court order on discovery.

An interior decorator asked a woodworker she met at a crafts fair to build a curly maple armoire. They entered into a written contract, with a contract price of $6,500 to be paid upon the decorator's receipt of the armoire. When the work was completed, the woodworker shipped the armoire to the decorator. After inspecting it, the decorator felt that it was not of the same high level of workmanship as she was expecting, given the other furniture that the woodworker had showcased at the fair, and a good faith dispute arose between the parties as to the workmanship. The decorator sent the woodworker a check for $4,000 marked "payment in full." The woodworker indorsed and cashed the check, then sued the decorator to recover the $2,500 balance. What would most courts likely hold? A The woodworker's cashing of the check constituted an accord and satisfaction, discharging the decorator's duty to pay the balance. B The woodworker can recover the $2,500 balance from the decorator. C The woodworker is estopped to sue for the balance because he cashed the check knowing that it was being tendered in full settlement. D The woodworker's indorsing a check so marked constituted a written release, thereby discharging the contract.

A Most courts would hold that there is a good faith dispute, and the check thus proposed an accord; the woodworker's act of cashing it is a satisfaction. A contract may be discharged by an accord and satisfaction. An accord is an agreement in which one party to an existing contract agrees to accept, in lieu of the performance that he is supposed to receive from the other party, some other, different performance. Satisfaction is the performance of the accord agreement. An accord and satisfaction generally may be accomplished by tender and acceptance of a check marked "payment in full" where there is a bona fide dispute as to the amount owed. Here, there is a good faith dispute between the parties as to the workmanship on the armoire. Therefore, the decorator's tender of the check marked "payment in full" and the woodworker's cashing of the check constituted an accord and satisfaction, discharging her duty to pay the balance.

During the course of their marriage, a husband told his wife that he stole a famous painting from a federal museum. Six months after the admission, the couple divorced. Shortly after the divorce, the husband was killed in an automobile accident. Later, the wife read in the paper that a man had been charged with the theft of the painting her husband admitted to stealing and was about to be tried in federal district court. She told her friend that the man was probably innocent because the husband told her that he had stolen the painting himself. The friend told several other people what the wife had told her, and eventually the story got back to the defense attorney. The attorney now wants the wife to testify in court to the husband's statement. Can the wife be compelled to testify? A Yes, but only because the husband is dead and cannot invoke his privilege. B Yes, because there is no privilege when the defendant is not a spouse. C No, because the couple was still married at the time of the disclosure. D No, because her testimony is not essential to prevent a fraud on the court.

A Similarly, if one spouse voluntarily reveals the contents of the communication to a stranger, that spouse waives the protection of the privilege as to herself (i.e., she cannot use the privilege to refuse to disclose, or to prevent another from disclosing, the communication), but the other spouse (i.e., the one who did not reveal the communication) retains this privilege. However, when the wife revealed to her friend what her husband had told her concerning the theft of the painting, the wife lost her privilege to refuse to disclose the matter. If the husband were alive, he would retain the privilege despite the wife's disclosure and could prevent her from testifying to his statement concerning the theft of the painting. Because the husband is dead, he cannot invoke his privilege. Because the wife has waived her privilege and the husband is unable to foreclose her testimony, she can be compelled to testify.

A victim and his former business partner, the defendant, had a bitter falling out after the victim accused the defendant of embezzling company funds. The defendant threatened to get even. Shortly thereafter, while driving on the expressway, a car swerved suddenly in front of the victim's car. Although the victim applied the brakes immediately, his car failed to stop. To avoid colliding with the car ahead of him, he swerved to the right and smashed into a concrete retaining wall. A passing motorist stopped and came to the aid of the victim. Bleeding profusely from a head wound, and rapidly losing consciousness, the victim said, "I don't think I'm going to make it. I tried to slow down, but my brakes didn't work. My former partner must have tampered with them to get back at me." With that, the victim lapsed into unconsciousness, and has been in a coma and on life support ever since. A personal injury suit has been filed on his behalf by a court-appointed guardian against the defendant. At trial, can the motorist testify as to the statement made by the victim? A No, because the victim did not know that the defendant tampered with the brakes. B No, because the victim is still alive. C Yes, because the victim thought he was about to die. D Yes, because this is a civil case.

A Testimony as to the statement made by the victim is inadmissible as a statement under belief of impending death, because the victim did not actually have firsthand knowledge that the defendant was responsible for the collision. the victim's statement represents a mere suspicion that the defendant tampered with the brakes. As well-founded as such a suspicion may be (given the history between the victim and the defendant), a statement based on mere suspicion rather than actual knowledge does not constitute a statement concerning the cause or circumstances of an "impending death" for purposes of the dying declarations exception.

An accountant employed by the Federal Communications Commission was offended by various jokes and cartoons that employees would post in the office cafeteria. The Commission did not have any rules regarding what employees could post in the cafeteria, and none of the cartoons were pornographic or harassing. Nevertheless, the accountant lodged a number of complaints with his supervisor that went unheeded. Finally, the accountant posted his own notice chastising the hypocrisy and immorality of the agency for allowing such cartoons when it was charged with ensuring a standard of decency on the public airwaves. The notice prompted a great deal of debate among employees and a great deal of displeasure on the part of the accountant's supervisor, particularly after it was posted on another employee's blog and received some media attention. A labor contract between the agency and the clerical workers' union contained a policy for providing for termination of union employees only for certain specified grounds, but the accountant was not a member of the union and was not covered by the policy or any other employment agreement. Which of the following statements is most accurate regarding the agency's right to dismiss the accountant? A The accountant has a liberty interest in the exercise of his First Amendment rights that entitles him to a hearing to contest the grounds of his dismissal. B The accountant has a property interest as a public employee that precludes him from being fired without notice and an opportunity to respond. C The accountant has no right to a hearing because his statements were not an expression of views on public issues. D The accountant has both a liberty interest and a property interest that entitles him to a pretermination evidentiary hearing.

A The accountant is entitled to a hearing because he has a liberty interest in the exercise of his First Amendment rights. If the accountant is fired, he has a right to a hearing to determine whether his First Amendment rights were violated by his dismissal. Under the Due Process Clause of the Fifth Amendment, a person has a liberty interest in the exercise of specific rights provided by the Constitution, including freedom of speech. A government employee may not be fired for expressing his views regarding public issues, but can be fired for speech that disrupts the employer's policies or undermines the employer's authority. Under the Court's expansive interpretation of what a public issue is in this context [see Rankin v. McPherson (1987)], the accountant's statement would probably qualify. At the very least, he can make enough of a showing that his termination violates his free speech rights to be entitled to a hearing on the issue under procedural due process principles.

A state statute provided for criminal penalties for "knowingly selling alcoholic beverages in violation of the regulations of the State Liquor Commission to any person under the age of 18." One of the State Liquor Commission regulations provided that "before an alcoholic beverage is sold to any person between the ages of 17 and 24, the seller must demand some form of photo identification to determine the buyer's age." A minor who looked much older than his age of 17 walked into a tavern located in the state and asked the bartender for a beer. The bartender never asked the minor for any form of identification, as he thought that he was at least 25 years old. Had the bartender asked for identification, the minor would have shown him a fake identification card showing that he was 21 years old. The bartender served the beer to the minor, who consumed it on the premises. The bartender was subsequently charged under the state statute for selling the beer to the minor. Is the bartender guilty? A No, because he reasonably believed that the minor was older than 25 years. B No, because the minor had fake identification with which he could have obtained the beer. C Yes, because he sold an alcoholic beverage to a minor, a strict liability crime. D Yes, because he failed to ask for identification, and the regulation does not provide for a mens rea requirement.

A The bartender's reasonable belief that the minor is 25 years old is a mistake of fact that negates the state of mind required by the statute. Ignorance or mistake as to a matter of fact will affect criminal guilt only if it shows that the defendant did not have the state of mind required for the crime. In addition, the mistake must be reasonable unless the offense is a specific intent crime. Here, the statute requires that the defendant have acted "knowingly" with respect to each of the material elements of the offense. A person acts knowingly with respect to the nature of his conduct when he is aware that his conduct is of that nature or that certain circumstances exist. At least one of the material elements of the offense here is that the sale be to a person under the age of 18. If the bartender believed that the minor was 25 years old, the bartender has not acted knowingly with respect to the fact that the purchaser was under 18, and he cannot be convicted of violating the statute.

The criminal statutes of the state define manslaughter and murder as they were defined at common law. As to insanity, the state has the following provision: "Under the defense of insanity a defendant may be entitled to acquittal if, because of mental illness, the defendant was unable to control his or her actions or to conform his or her conduct to the law." The defendant was put on trial in the state for the murder of his wife and her co-worker. The evidence at trial established that the defendant's wife was having an affair with the co-worker, and that the defendant learned of it and killed the pair. The defendant did not take the stand in his own defense. In his closing statement to the jury, the defendant's attorney made a statement, "Ladies and gentlemen, you must consider that there are some things that would provoke any one of us to kill, and there are things that make one unable to control one's actions." The defendant's attorney requested that the judge give the jury instructions on manslaughter and on insanity, and the judge agreed to do so. The judge also issued the following instructions: "INSTRUCTION #6: In order to mitigate an intentional killing to voluntary manslaughter, the burden of proof is on the defendant to establish that adequate provocation existed." "INSTRUCTION #8: Insanity is an affirmative defense and the burden of proof is on the defendant to establish that such insanity existed at the time of the killing." The jury found the defendant guilty of murder, and he appealed. He asserts that the jury instructions violated his rights under the federal Constitution. How should the appeals court rule? A Reverse the defendant's conviction, because Instruction #6 was improper. B Reverse the defendant's conviction, because Instruction #8 was improper. C Reverse the defendant's conviction, because both Instructions #6 and #8 were improper. D Uphold the defendant's conviction, because neither Instruction #6 nor Instruction #8 was improper.

A The court should reverse the defendant's conviction because Instruction #6 requires the defendant to disprove one of the elements of murder. Due process requires in criminal cases that the state prove guilt beyond a reasonable doubt. The prosecution has the burden of proving all of the elements of the crime charged. Thus, if malice aforethought is an element of murder and voluntary manslaughter is distinguished from murder by the existence of adequate provocation, the defendant cannot be required to prove that he committed the homicide in the heat of passion (i.e., with adequate provocation). Such a requirement would impose on the defendant the burden of disproving the element of malice aforethought, because "heat of passion" negates malice. Although the defendant can be given the burden of going forward with some evidence on the provocation issue, once he has done so, the prosecution bears the burden of proving that the killing was not done in the heat of passion. In the case at issue, Instruction #6 requires a defendant to prove that he committed the intentional killing under adequate provocation. At common law, and consequently in the state, malice aforethought is an element of murder. Therefore, this instruction in effect requires the defendant to disprove the element of malice aforethought, thereby relieving the state of its burden of proving all elements of the crime. As discussed above, such an instruction cannot pass constitutional muster. On the other hand, for an affirmative defense such as insanity, it is permissible to impose the burden of proof on the defendant.

A local entertainment section of a newspaper published a story on the town's business district, accompanied by photos of various businesses in the district. A minister who happened to be walking on the sidewalk in front of an adult bookstore when a photo was taken for the story became very upset when he saw it in the newspaper, because the camera angle made it appear that he was exiting the bookstore. If the minister sues the newspaper for invasion of privacy and establishes the above facts, is he likely to prevail? Yes, because the photo made it appear as if he was exiting an adult bookstore. B Yes, because the newspaper made a public disclosure of a private fact. C No, because he was on a public sidewalk when the photo was taken. D No, because he has not alleged any economic or pecuniary damages.

A The minister likely will prevail because unauthorized use of his picture that falsely makes him appear to be exiting the adult bookstore would be highly offensive to a reasonable person under the circumstances and constitute a false light invasion of privacy.

The owner of a house put the property up for sale. A surgeon entered into negotiations with the owner to purchase the house, and the parties agreed upon a sale price of $200,000 and a closing date. The owner told the surgeon that she would drop a contract in the mail and have her attorney draw up a deed. The owner signed a land sale contract, which included the property's address but did not contain a metes and bounds legal description. She mailed the contract to the surgeon that afternoon, although it was mailed too late for the last mail pickup of the day. The owner's attorney promptly drew up a deed and dropped it in the mail to his client. The surgeon received the contract the next day. After she mailed the contract, the owner received an offer of $250,000 for her property from her next-door neighbor, who wanted to expand beyond his own property line. The owner called her attorney and told him to inform the surgeon that the deal was off. The attorney e-mailed the surgeon, stating that his client had found another purchaser for the property, and that all matters regarding the surgeon's offer for the property were rescinded. The surgeon signed the contract anyway and returned it to the homeowner by registered mail. If the surgeon brings an action to compel the owner to convey the property to him for $200,000, is he likely to prevail? A Yes, because the owner signed the land sale contract. B Yes, because the contract was effective when the owner placed the document in the mail. C No, because the surgeon did not mail the signed contract until after he received the revocation by e-mail. D No, because the land sale contract does not contain the complete legal description of the property.

A The surgeon is entitled to specific performance because the owner signed the land sale contract. A contract was formed here when the parties orally agreed to the sale of the property. However, the contract was unenforceable at that time because, under the Statute of Frauds, a contract for the sale of land is unenforceable unless a memorandum containing the contract's essential terms is signed by the party to be charged. Here, the party to be charged is the owner, and she signed the land sale contract, a writing sufficient to satisfy the Statute of Frauds (a memorandum for the sale of land is sufficient if it contains the price, a description of the property-which need not be a "legal" description-and a designation of the parties). Thus, the contract was enforceable

Congress enacted a statute that purported to ban all discrimination against African-Americans in any commercial transaction taking place within the United States. Would the statute most likely be held constitutional? A Yes, under Thirteenth Amendment provisions barring badges or incidents of slavery. B Yes, because the federal government has an important interest in furthering the equal protection provisions of the Fourteenth Amendment. C No, because Congress's powers under the Commerce Clause do not extend so far as the statute would require. D No, because commercial transactions are not among the privileges or immunities of national citizenship.

A the Thirteenth Amendment prohibits slavery. The Enabling Clause of the amendment has been held to confer on Congress the authority to proscribe almost any private racially discriminatory action that can be characterized as a badge or incident of slavery. Because the statute at issue bans all discrimination against African-Americans in commercial transactions, it necessarily reaches private conduct. Such congressional action is constitutionally permissible pursuant to the Thirteenth Amendment.

A customer became ill from meat he purchased at the "The Corner Grocery" store in City A. The legal name of the store is "The Corner Grocery, Inc." The customer's attorney instituted a diversity action in federal district court, mistakenly naming as the defendant "The Corner Grocery" in City B, whose legal name is "The Corner Grocery Company, Ltd." The two are separate legal entities, but their stock is owned by the same persons in the same proportions. The complaint specifically described the store from which the pork was purchased, including its City A address. Approximately one month after filing the complaint, and after the statute of limitations had expired, the customer's attorney served process on an individual who is the president and 50% shareholder of both stores. The named defendant, the City B store, then filed its answer denying liability on the grounds that the customer never visited the City B store and never bought anything there. Upon learning that he had named the wrong store as defendant, the customer's attorney filed a motion to amend the complaint to substitute the name of the correct defendant for the incorrect defendant originally named. After the court allowed the amendment, the new defendant filed an answer asserting that the claim against it was barred by the statute of limitations. In response, the customer's attorney asserted that the amended complaint is not barred because it asserts a claim that arises from the same transaction as the original complaint and therefore relates back to the time of the original complaint. Is the amended complaint barred even though it relates back to the time of the original complaint? A No, because the customer appears to have acted in good faith and exercised reasonable care in naming the proper defendant. B No, because the new defendant received timely notice of the action so as not to be prejudiced in defending the merits and knew or should have known that the action would have been brought against it but for a mistake concerning the proper party's identity. C Yes, because the proper defendant was not served with process within the time prescribed for service of the original complaint. D Yes, because both the original and the amended claims are barred by the statute of limitations since service of process did not occur until after the statute of limitations expired.

B

A landlord owned a prestigious downtown office building. A law firm leased the entire building from the landlord for a term of 20 years. The lease included a provision that taxes on the building would be paid by "the lessee, his successors, and assigns." The law firm occupied the building and paid the rent and taxes for eight years. At the end of the eight-year period, the law firm assigned the balance of the lease to an accounting firm and vacated the premises. The assignment was written, but there was no provision concerning the accounting firm's assumption of the duties under the lease. The accounting firm occupied the building and paid the rent and taxes for five years. At the end of the five-year period, the accounting firm subleased the building for five years to an investment company and vacated the premises. The sublease was written, but there was no provision concerning the investment company's assumption of the duties under the lease. The investment company now occupies the building and has paid the rent but not the taxes. The landlord has sued all three (i.e., the law firm, the accounting firm, and the investment company) for failure to pay the taxes. The landlord should prevail against whom? A The law firm only. B The law firm and the accounting firm, but not the investment company. C The accounting firm and the investment company, but not the law firm. D The law firm, the accounting firm, and the investment company.

B

A plaintiff sued an auto manufacturer for negligence after a car accident involving the plaintiff's car that was made by the auto manufacturer. Sixty days after service of the complaint and 40 days after service of the manufacturer's answer that contained no counterclaim, the plaintiff filed a motion seeking to file an amended complaint adding a claim for strict products liability against the auto manufacturer stemming from the same incident. The statute of limitations for strict products liability claims expired one week before the motion was filed. How should the court rule on the plaintiff's motion? A Grant the motion, because every party is entitled to amend once as a matter of course. B Grant the motion, because the amended complaint relates back. C Deny the motion, because, while the motion is timely, the proposed claim is futile because the statute of limitations has run. D Deny the motion, because it is not timely.

B

A retailer entered into a written contract with a wholesaler whereby the wholesaler agreed to sell, and the retailer agreed to buy, 100 boxes of sunglasses manufactured by a large corporation located in a neighboring city. The agreed-upon price was $75 per box. Two weeks before the specified delivery date, the wholesaler told the retailer that it would not be able to fill its order, because of unexpected high demand for sunglasses this season. Although the retailer learned that the needed quantity of the same brand of sunglasses could be shipped within two days for $83 per box from a supplier in another area, the retailer instead purchased 100 boxes of the sunglasses locally at a cost of $90 per box. These sunglasses were of a slightly higher quality than the sunglasses that were originally contracted for. A few days before the original delivery date, the wholesaler notified the retailer that it would fill the order, and tendered 100 boxes of the sunglasses on the date of delivery. However, the retailer refused to accept them. At that time, the wholesale market price of the sunglasses had declined to $80 per box. If the retailer sues the wholesaler for damages based on the wholesaler's alleged breach, what is the retailer likely to recover? A $1,500, the difference between the cost of cover and the contract price. B $800, the difference between the contract price and the nonlocal supplier's price. C $500, representing the difference between the contract price and the wholesale market price at the time of performance. D Nothing, because the retailer obtained cover without waiting a commercially reasonable time for the wholesaler to retract the repudiation.

B

A young teenager pointed a squirt gun at an older teenager as if she was going to squirt him, although the younger teenager knew that the gun was empty. The older teenager did not know that the gun was empty and yelled, "A little water isn't going to hurt me." The younger teenager pulled the trigger and yelled back, "You're lucky, it wasn't even loaded." Is the younger teenager liable to the older teenager? A Yes, because the younger teenager attempted a battery. B Yes, because the younger teenager committed an assault. C No, because the squirt gun was not loaded. D No, because the older teen was not worried about getting wet.

B

At the defendant's trial for murder, facts were introduced that the defendant acted in the heat of passion. After a lengthy trial, the defendant was convicted of manslaughter. On appeal, the conviction was reversed on procedural grounds. The state immediately moved to retry the defendant, again bringing murder charges against her. The defendant moved to strike the murder charge, and the court refused to grant the motion. After the second trial, the defendant was again convicted of the lesser charge of manslaughter. The defendant appeals the second conviction, claiming that it violated her constitutional rights. May the second conviction stand? A No, because the state could not refile charges after the acquittal. B No, because the state could not retry the defendant for murder under the circumstances. C Yes, because the state could retry the defendant because the manslaughter conviction was overturned. D Yes, because the defendant was reconvicted of the lesser charge again, so any error was harmless.

B

While at a party, the defendant ran into an acquaintance. The acquaintance proceeded to ridicule the defendant about his looks. After an hour of verbal abuse by the acquaintance, the defendant suddenly took a champagne bottle that was on a nearby table and struck the acquaintance over the head, killing him instantly. At his arrest, the defendant told the police that voices inside his head told him to shut the acquaintance up, permanently. The defendant was tried in a jurisdiction that follows the Model Penal Code test for insanity. At trial, the defendant's lawyer introduced psychiatric testimony indicating that the defendant suffered from a mental illness. Which of the following, if proved by the defense, would most likely relieve the defendant of criminal responsibility? A The defendant's actions were a product of his mental illness. B The defendant could not appreciate the criminality of killing the acquaintance, or he could not conform his conduct to the requirements of the law. C The defendant did not know that killing the acquaintance was wrong, or he could not understand the nature and quality of his actions. D The defendant was unable to control himself or conform his conduct to the law.

B

A homeowner and a builder entered into a written contract to build a sauna in a spare room in the homeowner's home at a cost of $3,000. The contract contained a clause stating that the builder will not begin construction without prior approval of the plans by the homeowner's certified public accountant. The builder submitted his designs to both the homeowner and the accountant. The homeowner liked the plans, but the accountant did not and withheld his approval. The builder asked the homeowner whether she wanted him to submit new designs. The homeowner told the builder orally, "No! Your designs are great! My accountant is crazy! You go right ahead and construct the sauna." The builder constructed the sauna. The homeowner now refuses to pay the builder, citing the clause requiring approval by the accountant. If the builder sues the homeowner, what will the builder likely recover? A The full contract price, because the accountant's approval was not a condition precedent for the contract to take effect. B The full contract price, because once the builder began building the sauna after speaking to the homeowner, the homeowner did nothing to stop the builder. C The reasonable value of the builder's services and materials, because otherwise the homeowner would be unjustly enriched. D Nothing, because the homeowner's oral statement will be excluded by the parol evidence rule.

B By her statement to the builder, the homeowner waived the benefit of the condition requiring the accountant's approval of the design plans, and the builder detrimentally relied on the statement by building the sauna. Thus, there is a binding waiver of the condition.

After picking up a load of hazardous chemical waste, a truck driver for a waste management company set out on the road to his next stop. However, he had failed to secure the latch on the back panel of the truck. Consequently, the panel opened while the truck was on the road, and a metal canister full of chemical waste fell onto the road. A car struck the canister, causing the car to veer off the road and injure the driver. The driver filed suit against the company for his injuries. The jurisdiction in which the above events took place has adopted a rule of partial comparative negligence. At trial, the driver of the car admitted that he had momentarily taken his eyes off the road to look at his speedometer. When he had looked up again, the canister was there and he could not stop in time. The jury found that the company, through its truck driver, had acted willfully and wantonly and was 90% at fault, while the driver of the car was 10% at fault. The driver filed a motion for judgment notwithstanding the verdict, seeking recovery for 100% of his damages. If the judge grants the motion, what is the most likely reason? A A plaintiff's comparative negligence is not taken into account in cases of willful and wanton conduct by the defendant. B A state ordinance mandating motorists to stay within the posted speed limit requires as a matter of law an occasional glance at the speedometer. C The company was more than 50% at fault. D The company was engaged in an abnormally dangerous activity.

B If the driver was effectively required by statute to take an occasional quick look at his speedometer to make sure that he was complying with appropriate speed limits, then his momentary glance at the speedometer in the instant case would, as a matter of law, not constitute negligent conduct. Because this is a matter of law, the judge would be authorized to correct this aspect of the jury's verdict. If the driver is thus found to be not negligent in this matter, his recovery will not be reduced.

A man purchased a large flat screen television and decided to mount it on the ceiling over his bed. The manual that came with the product included detailed instructions and illustrations on how to mount the television on different types of walls, along with all the required hardware, but contained neither instructions nor warnings regarding mounting on the ceiling. The man carefully followed the wall-mounting instructions and was satisfied that it would hold. In fact, however, the mounting was not appropriate for ceilings. The next night, a woman who was the man's overnight guest was seriously injured when the television came loose and fell on the bed. Will the woman prevail in a suit against the company that manufactured the television? A Yes, because the manufacturer had a duty to include warnings for all potential placements of its product. B Yes, if the manufacturer knew that its television was sometimes mounted on ceilings rather than walls. C No, if the manufacturer's manual had all of the customary warnings for this type of product. D No, because the man was negligent in mounting the television on the ceiling.

B Knowledge on the part of the manufacturer that its television was being mounted on the ceiling would give rise to a duty to include in the manual warnings against the practice or detailed instructions on how to safely mount it. The television hardware and instructions were appropriate for its intended mounting on the wall. However, courts in a strict liability case require a commercial supplier to anticipate reasonably foreseeable uses even if they are misuses of the product. If the manufacturer knew that members of the public were sometimes mounting the television on the ceiling, marketing the product without including either warnings against the practice or appropriate hardware and instructions on how to safely do so made the product so defective as to be unreasonably dangerous if it were improperly mounted. Under a strict liability theory, the manufacturer is liable for supplying a defective product. As a guest of a purchaser of the product, the woman is a foreseeable plaintiff; thus, the manufacturer may be liable to her. The defective product actually and proximately caused the woman to suffer serious injuries. Therefore, the manufacturer is liable to the woman in a strict products liability action.

A police officer saw a car containing three teenagers driving slowly down the street at 1 a.m. She waited for it to go by her and, after it was far enough ahead, started to follow it. Several blocks later, the car rolled through a stop sign. The officer immediately pulled the car over and requested the driver's license. A license check showed that the driver had five outstanding parking tickets. A statute in the jurisdiction permits an arrest to be made if a driver has four or more outstanding parking or traffic violations. The officer decided to take the driver in on the tickets. She informed the driver that he was under arrest and asked him to step out of the car. When the driver got out, the officer patted him down and found a gun in his waistband. Calling for backup, she decided to haul all three teenagers to jail. Subsequent testing showed that the gun had been used in a recent homicide during a store robbery by three young men. One of the passengers made a motion to prevent the introduction of the gun at his trial for murder and robbery. How should the judge rule? A Deny the motion, because the gun was found after the driver had been arrested. B Deny the motion, because the officer lawfully stopped the car. C Grant the motion, because the officer had no valid reason to be following the automobile. D Grant the motion, because the officer had not arrested the driver for suspicion of robbing the store or committing the homicide.

B READ THE QUESTION CORRECTLY

A landlord entered into a written lease of a bakery for a term of 25 years with a baker. The parties agreed to a right of first refusal if the bakery was offered for sale during the term of the lease. The lease also permitted assignments and subleases on notice to the landlord. Three years later, the baker retired and, after notifying the landlord, transferred the lease to a chocolatier. Twenty-one years later, the landlord entered into a contract with a buyer for the sale of the bakery for $100,000. The landlord had informed the buyer of the lease but had forgotten about the right of first refusal. When the chocolatier learned of the sale to the buyer, she informed both the landlord and the buyer that she wanted to exercise her option and was prepared to purchase the bakery for the contract price. The jurisdiction's Rule Against Perpetuities is unmodified by statute. Can the chocolatier enforce the option? A Yes, because an option held by a tenant on leased property cannot be separated from the leasehold interest. B Yes, because the option touches and concerns the leasehold estate. C No, because the transfer to the chocolatier made the option void under the Rule Against Perpetuities. D No, because the option was not specifically included when the lease was transferred to the chocolatier.

B The chocolatier can enforce the option to purchase because it is a covenant that runs with the land. When a tenant makes a complete transfer of the entire remaining term of his leasehold interest, it constitutes an assignment. The assignee and the landlord are then in privity of estate, and each is liable to the other on all covenants in the lease that run with the land. The covenant runs with the land if the original parties so intend and the covenant "touches and concerns" the leased land, i.e., burdens the landlord and benefits the tenant with respect to their interests in the property. (A) is incorrect because most courts do not bar an option from being separated from the leasehold interest if that is the parties' intent. The tenant may transfer the leasehold interest while retaining the option to purchase, or vice versa. Whether the option in this case stayed with the leasehold interest depends on whether it was a covenant that runs with the land.

A sailing enthusiast went to a boat builder and told him that he wanted a yacht built to his specifications. They agreed that the price would be $400,000, and that the sailing enthusiast was to make payment in full within 30 days after he had accepted delivery of the yacht. They further agreed that the boat builder would not subcontract any of the work. The boat builder, however, contacted a master sail maker and subcontracted the sails for the yacht to him. They agreed orally that the boat builder would pay the sail maker $25,000 for the sails within 20 days of receiving them. The boat builder did not tell the sail maker of his agreement with the sailing enthusiast regarding subcontracting. The sail maker made the sails and delivered them to the boat builder, who then completed the yacht and delivered the boat to the sailing enthusiast. Although the yacht was built to his specifications, the sailing enthusiast refused to accept it after he learned that the boat builder had subcontracted for the sails. When the 20-day payment period for the sails had expired, the sail maker went to the boat builder and demanded the $25,000. The boat builder told the sail maker that he could not pay the $25,000 unless the sailing enthusiast paid him for the yacht. If the sail maker brings an action against the boat builder for breach of contract, is he likely to prevail? A Yes, because both parties are merchants. B Yes, because the sail maker fully performed. C No, because the boat builder had agreed not to subcontract. D No, because their agreement was oral.

B The contract between the sail maker and the boat builder is enforceable despite the Statute of Frauds because the sail maker fully performed. The contract here was for the sale of goods (sails) for the price of $500 or more; thus, the contract is within the Statute of Frauds. A contract within the Statute of Frauds is generally unenforceable absent written evidence of a contract signed by the party to be charged. However, there is an exception to the general rule for goods received and accepted. Here, although the contract was oral, the boat builder accepted the sails, and so he is bound despite the Statute. Note that the boat builder might also be bound under another exception to the Statute-for specially manufactured goods if the sails were made specially for the yacht and were not suitable for sale to others.

The defendant is on trial for fraudulently signing a check for $10,000. The defendant has denied that she signed the check. The prosecutor calls the landlord of the apartment building in which the defendant has resided for three months before her arrest. The landlord intends to testify that it is the defendant's signature on the check, and he bases his opinion of the authenticity of her signature on the ground that he saw her sign the lease to his apartment. Should the trial court find this testimony admissible? A Yes, because there was only a short period of time between when the landlord saw her sign the lease and the time of trial. B Yes, because the landlord has previously seen the signature. C No, because the landlord has seen the signature only once and is not acting as a handwriting expert. D No, because the testimony is inherently unreliable.

B The court should find this testimony admissible. Any person can testify to the authenticity of another's signature as long as that witness has seen the person's signature and can express an opinion regarding its authenticity. The only restriction is that a nonexpert cannot become familiar with the handwriting merely for the purpose of testifying, and this was not the case here. There is no requirement that the witness have seen the signature recently;

A wife is on trial for the murder of her husband. She is accused of pushing him from the window of their 13th floor apartment; she claims he committed suicide. The wife called an operator for a suicide-prevention clinic to testify that the deceased husband had called the clinic on more than one occasion, each time telling the operator that he wanted to "end it all." Is the testimony admissible? A Yes, because the statements were made in "contemplation" of death. B Yes, because it tends to show that the husband intended to commit suicide. C No, because it violates the psychiatrist-patient privilege. D No, because no phone calls were made to the clinic by the husband on the day he died.

B The court should rule that the testimony is admissible. Under the state of mind exception to the hearsay rule, a declaration of intent to do something in the future is admissible as circumstantial evidence tending to show that the intent was carried out. Here, the husband's statements to the operator tend to show that the husband intended to commit suicide, so they are admissible to prove that he did so.

A driver and his passenger were involved in an automobile accident when the driver ran a red light and crashed into another car. Due to a manufacturing defect in the automobile's airbag system, the passenger side airbag did not deploy. The passenger was killed on impact. The passenger's estate brought suit against the driver and the airbag's manufacturer. At trial it is established that the driver was negligent in running the red light. What effect would such proof have on the claim of the passenger's estate against the airbag manufacturer? A It would reduce recovery by the estate if the action against the manufacturer is based on negligence. B It would bar recovery by the estate if the trier of fact finds that the driver was the sole legal cause of the passenger's death. C It would bar recovery by the estate if it is shown that the driver is the sole legal heir of the passenger's estate. DIt would have no effect on recovery by the estate as long as the action against the manufacturer is based on strict liability.

B The driver's negligence would bar recovery if it was the sole legal cause of the passenger's death. Regardless of the theory that the plaintiff is using in a products liability action, actual and proximate cause must be established. If the driver's negligence is the sole legal or proximate cause of the passenger's death, it would preclude the estate's suit against the airbag manufacturer because the defect was not a legal cause of the passenger's death.

On November 5, an athletic shoe manufacturer entered into a written contract with a shoe store to carry its running shoes. The contract included a provision that "5% of the proceeds attributable to the sale of the manufacturer's shoes by the shoe store during the month of February (American Heart Month) each year would be donated to the local hospital's cardiovascular wing." The day after the parties signed their contract, the store owner informed the hospital of the planned donation and indicated that the hospital could expect to receive about $1,500 in early March. In anticipation of the donation, the hospital purchased a new heart monitor on January 5. Because the Christmas sales season had been poor, on January 15 the manufacturer and the shoe store agreed to modify their contract to eliminate the provision for payments to the hospital's cardiovascular wing. When the hospital did not receive the payment in March, it discovered that the parties had modified the agreement. If the hospital brings an action against the shoe store to recover 5% of the proceeds from the sale of the shoes, is it likely to prevail? A Yes, because the hospital's rights vested when it learned of the original agreement. B Yes, because the hospital detrimentally relied on the parties' agreement. CNo, because the hospital's rights had not vested when the modification was made. D No, because the payment is a gift, and the hospital is a donee-beneficiary.

B The hospital is likely to prevail because it relied to its detriment on the parties' agreement. An intended third-party beneficiary can enforce a contract only after its rights have vested. Vesting occurs when the beneficiary: (i) manifests assent to the promise in a manner invited or requested by the parties; (ii) brings suit to enforce the promise; or (iii) materially changes position in justifiable reliance on the promise. Here, the hospital purchased the heart monitor in detrimental reliance on the agreement between the manufacturer and the shoe store. The reliance was reasonable because the store owner informed the hospital of the agreement, the approximate amount that they would receive, and the date they would receive it. Once the hospital's rights vested, the manufacturer and the store could not vary the hospital's rights without the hospital's consent. (A) is incorrect because, as noted above, the third-party beneficiary's rights do not vest until occurrence of one of the specified conditions. Vesting does not automatically occur on execution of the contract or when the beneficiary learns of the promise.

A state enacted a law banning the use within the state of computerized telephone solicitation devices, and requiring that all telephone solicitation calls within the state to in-state numbers be dialed by human beings. Federal legislation and administrative regulations control only the rates to be charged for telephone calls. Is the legislation valid? A Yes, because it involves wholly intrastate commerce that is not subject to federal regulation. B Yes, because the statute does not conflict with federal legislation or the negative implications of the Commerce Clause. C No, because the statute is preempted by federal legislation under the Supremacy Clause. D No, because the statute is an unconstitutional attempt by a state to regulate interstate commerce.

B The states may regulate local aspects of interstate commerce as long as Congress has not adopted regulations concerning the subject matter or preempting the entire area of regulation. Even absent federal legislation, under the negative implications of the Commerce Clause the state regulation must not discriminate against interstate commerce or unduly burden it. Here, there is no federal legislation directly conflicting with the state law, as the facts state that the federal government only regulates the rates that may be charged by phone companies. Neither has Congress preempted the field.

On March 1, a health food store owner and a health food distributor entered into a written agreement providing that the distributor would supply the store owner with his natural foods requirements over the next 12 months. In return, the store owner agreed to purchase only from the distributor. The agreement also provided that payment for any purchases made under the agreement during the month of April would be turned over to a nonprofit food research corporation, "to carry on its good works." On April 1, the store owner ordered and the distributor shipped 20 cases of health food products, at a wholesale price of $4,000, which remains unpaid. On April 15, the store owner sold his store, inventory, and accounts receivable to a chain operation that also sells health food products. As part of the sale, the store owner assigned to the chain operation the contract with the distributor. The chain operation promptly notified the distributor of the sale and assignment. The $4,000 for the April 1 transaction remained unpaid and, on May 31, the food research corporation commenced suit to collect the money. Against whom may the action be maintained? A The store owner only. B Both the store owner and the chain operation. C The chain operation only. D Neither the store owner nor the chain operation.

B The store owner (the promisor) remains liable based on his contract with the distributor (the promisee), and the chain operation is liable based on its assumption of the contract with the distributor. The food research corporation is expressly designated in the contract between the store owner and the distributor as a party to whom payment for any April purchases is to be directly made. Thus, the portion of the contract providing for payment is primarily for its benefit. Consequently, the food research corporation is an intended third-party beneficiary of the store owner's promise to make the April payment. As the third-party beneficiary, it has a right of action against the promisor (the store owner) for enforcement of the promise to pay. When the store owner assigned to the chain operation his contract with the distributor, the chain operation was deemed to have assumed the store owner's duties under that contract as well as being assigned the rights thereunder. Thus, the chain operation can be held to the duty to pay for the health food products that were ordered in April. Although the chain operation is deemed to have assumed the duties of the store owner under the contract, the store owner (the delegator) remains liable on the contract. Consequently, the food research corporation has a right of action for the April payment against both the store owner and the chain operation (although it will be limited to only one recovery, see below).

A state legislature enacted a statute providing for loaning certain textbooks on secular subjects to students in all public and private schools. In accordance with the statute, the state board of education distributed textbooks to a private school that offered religious instruction and admitted only Caucasian students. Which of the following is the strongest argument against the constitutionality of free distribution of textbooks to the students at the private school? A A state may not constitutionally aid private schools through distribution of textbooks. B Segregation is furthered by the distribution of textbooks to these students. C The distribution of textbooks advances religion because it is impossible to separate their secular and religious uses. D The distribution of textbooks fosters excessive government entanglement with religion.

B The strongest argument is that state provision of textbooks to the segregated private school violates the Equal Protection Clause by giving state support to a racially segregated educational process. This may not be a winning argument but it is clearly the best of the lot. (C) and (D) are parts of the test for violation of the Establishment Clause. The Supreme Court has held that a state lending textbooks on secular subjects to all students, including those at religious schools, does not violate the Establishment Clause. Thus, (C) and (D) are incorrect.

As a result of a personal injury lawsuit, a victim obtained a judgment against a tortfeasor for $100,000. The tortfeasor, who had few assets, did not pay the judgment. On April 1 of the following year, the tortfeasor inherited a parcel of land from her uncle. On May 1, the tortfeasor entered into a contract with a buyer to sell the land for $120,000. The contract was not recorded. The buyer immediately applied to a bank for a loan of $100,000. The bank approved the buyer's loan, and on May 15, a closing was held. The tortfeasor deeded the land to the buyer, and the buyer executed a mortgage for $100,000 to the bank. Due to an error by the title company, the deed from the tortfeasor to the buyer was not recorded, although the mortgage to the bank was recorded. Neither the buyer nor the bank had any knowledge of the victim's judgment. On May 20, the victim recorded his judgment in the county recorder's office where the land was located. At that time, he had no knowledge of the buyer's or the bank's rights. When he learned about them, he immediately brought a proceeding to foreclose his judgment lien, naming the tortfeasor, the buyer, and the bank as parties. The jurisdiction has a typical grantor/grantee recording index, and has enacted the following statute: "Any judgment properly filed in the county recorder's office shall, for 10 years from filing, be a lien on the real property then owned or subsequently acquired by any person against whom the judgment is rendered. No conveyance or mortgage of real property shall be good against subsequent bona fide purchasers for value and without notice unless the same be recorded according to law." As between the victim and the bank, which party's interest in the land will be given priority? A The bank, because the bank recorded its mortgage before the victim recorded his judgment lien. B The bank, because the victim is not protected by the recording statute. C The victim, because the victim's judgment was filed in the recorder's office before the buyer's deed was recorded. D The victim, because the judgment lien extends to after-acquired property.

B The victim will not likely prevail against the bank because a majority of courts hold that the judgment lienor is not protected by the recording statute. If the statute here, which is a notice statute, were applicable to protect the victim, he would have priority over the bank because his judgment lien was recorded before the buyer's deed was recorded. Under this view, the bank's mortgage would have been considered "wild" and would be deemed unrecorded because the preceding conveyance, the buyer's deed, was actually unrecorded. A searcher in the public records would therefore have been unable to find the mortgage. Hence, if the statute were applicable to protect the victim, he would have priority over the bank. However, most courts reason that either (i) a judgment creditor is not a bona fide purchaser because he did not pay contemporaneous value for the judgment, or (ii) the judgment attaches only to property "owned" by the debtor, and not to property previously conveyed away, even if that conveyance was not recorded. Under the statute in the present question, a judgment does not attach until it is recorded. Here, the victim's judgment did not attach to the land until after the bank obtained a mortgage on it, and the recording statute does not change that result. The failure of the buyer to record, and the resultant treatment of the bank as unrecorded, is irrelevant. Thus, the bank's mortgage is superior to the victim's lien.

A jogger not paying attention to where he was going collided with an elderly woman in a wheelchair, injuring her. The woman, who was mentally incapacitated, was being pushed down the sidewalk by her daughter, who was also her legal guardian. After the accident, the daughter did not take the woman to the doctor for treatment because the daughter had an unreasonable fear of catching a virus at the doctor's office. As a result, the woman's wounds became infected and took longer to heal. What argument provides the woman with her best chance to recover against the jogger for all of her injuries? A The doctrine of mitigation of damages does not bar recovery for the original injuries caused by the jogger. B Any unreasonable conduct on her daughter's part is not to be imputed to the woman. C Neither the woman nor her daughter were contributorily negligent in causing the collision. D Defendants must take their victims as they find them, including the unreasonable behavior of their legal guardians.

B The woman's best argument is that her daughter's refusal to take her to a physician, if deemed to be negligent, is not imputed to her. A plaintiff has a duty to take reasonable steps to mitigate damages. Thus, in personal injury cases, there is a duty to seek appropriate treatment to effect healing and to prevent aggravation. Failure to do so will preclude recovery for any particular item of injury that occurs or is aggravated due to the failure to mitigate. Thus, the woman's not consulting a doctor could limit her recovery to the damages for the original injury only. However, she is incapacitated and in the care and control of a legal guardian (her daughter), who decided not to bring her to a doctor. In actions against a third party, a legal guardian's negligence is not imputed to the person under her care.

A seller entered into a written contract to sell a tract of land to a buyer. The buyer was to pay $1,500 per month for five years, at which time the seller would deliver a warranty deed. The contract was silent as to the quality of title to be conveyed. After making 12 payments, the buyer discovered that a neighbor had an easement of way over the land, which was not discussed at the time the seller and buyer entered into the contract. The neighbor had not used the easement over the previous year because she had been out of the country. On the basis of the easement, the buyer wishes to cancel the contract. Which party is more likely to prevail? A The seller, because the neighbor's easement has been extinguished. B The seller, because the buyer has no basis on which to rescind the contract. C The buyer, because the obligation to convey marketable title is implied. D The buyer, because the seller has breached the covenant against encumbrances.

B Where an installment land contract is used, the seller's obligation is to furnish marketable title when delivery is to occur, e.g., when the buyer has made his final payment.

A comprehensive federal health-care reform statute created a Federal Health Policy Board, which was directed to monitor the fees charged for various medical procedures covered by insurance. The board also had the power to subpoena records to determine whether fee increases were a true reflection of cost increases. Nothing in the statute provided for caps on fee increases. Because of the continuing escalation of health-care costs while the statute was being debated, several states had passed health-care legislation on their own. One state passed legislation that prohibited most fee increases of 10% or more per year for specified health-care services covered by insurance, and created a health-care review board to regulate these costs and impose monetary penalties on health-care providers or insurers that tried to circumvent the cap. Which of the following would be the best basis for finding the state provision unconstitutional? A The federal legislation was passed after the state legislation and therefore supersedes it. B The Federal Health Policy Board was constituted with many of the same powers as the state board but was not given the power to impose sanctions. C The state provision impairs existing contracts between health-care providers and insurers in violation of the Contract Clause. D Health-care fee caps create an undue burden on interstate commerce even in the absence of federal regulation.

B Where the federal laws are comprehensive or a federal agency is created to oversee the field, preemption will often be found. The fact that the health-care legislation was comprehensive but the federal board was not given regulatory or enforcement power suggests that Congress did not want specific restrictions in these areas and may have wanted free-market principles to determine fee increases at the outset. The state board's power to impose these restrictions may violate the Supremacy Clause under these circumstances.

A researcher and an analyst had employment contracts with a corporation. The corporation fired them because it believed they had conspired to give its trade secrets to a competitor. Knowing that the conspiracy allegations were not true, the researcher and the analyst sued the corporation in federal district court for breach of their employment contracts. The researcher seeks $100,000 in damages and the analyst seeks $50,000. They are both citizens of State A. The corporation is incorporated in State B, and its principal place of business is in State B. Does the federal district court have subject matter jurisdiction over both of the claims? A Yes, the court has diversity of citizenship jurisdiction over both claims because the amounts of their related claims may be aggregated to satisfy the amount in controversy requirement. B Yes, the court has diversity of citizenship jurisdiction over the researcher's $100,000 claim and supplemental jurisdiction over the analyst's $50,000 claim. C No, the court does not have subject matter jurisdiction over the analyst's $50,000 claim because the claims of both plaintiffs may not be aggregated to satisfy the amount in controversy requirement. D No, the court does not have subject matter jurisdiction over either claim because the breach of contract claim is based on state law.

B a co-plaintiff with a below-limit claim generally can use supplemental jurisdiction to have his claim heard in federal court so long as the co-plaintiff's presence in the suit does not destroy complete diversity.

A father was angry at his son's coach because the coach would never let the son into a game. In order to exact revenge, the father decided to plant an incendiary device on the coach's front porch. The father believed the device would start a fire that would destroy the coach's home and perhaps injure him as well. However, the father made a mistake while assembling the incendiary device, and it was impossible for the device to do any harm. When the device went off, it did nothing more than produce a foul odor. If the father is charged with attempted murder and attempted arson in a common law jurisdiction, which of the following decisions is most likely to be reached by the court? A The father is guilty of attempted murder and attempted arson. B The father is guilty of attempted murder, but he is not guilty of attempted arson. C The father is not guilty of attempted murder, but he is guilty of attempted arson. D The father is not guilty of attempted murder or attempted arson.

C

A motorist driving home one night on a desolate two-lane road stopped when he saw a person lying on the road next to a bicycle. The cyclist had slipped and fallen off his bicycle, and was knocked unconscious when he hit his head on the pavement. Not wishing to get involved and seeing that no one else was around, the motorist got back into his car and drove away without making any effort to help the cyclist, even though he had a cell phone with which he could have summoned aid. The cyclist remained lying in the same place and was later struck by another car. If the cyclist brings suit against the motorist for injuries suffered when he was struck by the other car, will the cyclist prevail? A Yes, because a reasonable person under the same circumstances would have come to the cyclist's aid.Incorrect B Yes, because by stopping and examining the cyclist, the motorist, as a matter of law, assumed a duty to aid him. C No, because the motorist was not responsible for causing the cyclist to be lying by the side of the road. D No, because the cyclist's injuries were caused by the negligence of another driver.

C

A woman purchased a tract of land from a man by warranty deed. Unbeknownst to the woman, the man was not the actual owner of the tract. The woman built a home on the tract and moved into it. Two years later, the actual owner learned of the man's transaction with the woman and prevented the woman from entering the tract from that point forward. This led to a costly court battle. When the woman notified the man and told him that she thought it was his duty to straighten this out, he ignored her. The statute of limitations for actions on deed covenants is fours years. The woman would succeed in a suit for damages against the man for breach of which of the following covenants of title? A The covenant of quiet enjoyment only. B The covenants of seisin, right to convey, quiet enjoyment, warranty, further assurances, and the covenant against encumbrances. C The covenants of seisin, right to convey, quiet enjoyment, warranty, and further assurances. D The covenants of seisin and right to convey only.

C

After the release of various news stories about the President's possible violation of political campaign funding laws, a federal grand jury investigation and an investigation by a special Senate subcommittee were initiated. The Senate subcommittee subpoenaed personal records of the President from several top officers of the executive branch. Learning of the subpoenas, the President ordered all executive officials to refuse to turn over materials, claiming "executive privilege." Which of the following statements is most accurate? A The subpoena violates the constitutional principle of separation of powers. B The President's executive privilege is absolute, except in cases of impeachment. C The presidential papers are presumptively privileged, but the privilege must yield to a demonstrated legitimate need for evidence in a pending legislative proceeding. D The President's executive privilege applies to proceedings by Congress, but not to proceedings by the courts.

C

An officer on routine patrol noticed a flashlight moving within a darkened house and stopped to investigate. The suspect, who had broken into the home to steal valuables, caught sight of the patrol car, dropped the bag of valuables as he was about to carry them out of the house, and tried to sneak out the back way. The officer saw him sneaking out and seized him. The suspect, who had a lock-picking device in his possession, pulled out two $100 bills from his wallet, stating that he did not take anything and would like to forget the whole thing. The officer took the money, stating that she would give him a break this time around, and let the suspect go. How may the officer be charged in this situation? A As an accessory after the fact to burglary and larceny. B As an accomplice to the crimes of burglary and larceny. C As an accessory after the fact to burglary only. D Neither as an accomplice nor an accessory after the fact.

C

To reduce deer overpopulation in state forests, state Blue adopted a statute allowing anyone with a valid deer hunting license from any state to hunt deer within state Blue. The act also imposed a $0.25 per pound tax on each deer killed within the state. Funds from the tax were earmarked to support state forest land. State Red is adjacent to state Blue and also has an overabundance of deer. To encourage hunting, state Red does not impose a tax on deer taken from its forests. A hunter who is a resident of state Red and who is licensed to hunt there earns his living by supplying wild game to several high-end restaurants in state Red. While legally hunting deer within state Red, the hunter inadvertently crossed the state line and killed a deer in state Blue. Upon hearing the hunter's shot, a state Blue game warden arrived at the scene, approximated the weight of the kill, and handed the hunter a tax bill based on the approximation. The bill provided a method for challenging the approximated weight of the deer, but the hunter refused to pay any tax on his kill. He instead filed suit in federal court to enjoin collection of the state Blue tax on constitutional grounds. Which of the following results is most likely? A The hunter will prevail because the tax is invalid under the Commerce Clause. B The hunter will prevail because the tax is invalid under the Interstate Privileges and Immunities Clause of Article IV, Section 2. C State Blue will prevail because the tax is valid under the Commerce Clause. D State Blue will prevail because the tax is valid under the Import-Export Clause.

C

To stabilize state corn prices, a state purchased large quantities of corn from resident farmers and converted the corn into biodegradable plastics. The state then sold the plastics to state residents at cost and to out-of-state residents at cost plus 25%. An out-of-state corporation purchased biodegradable plastics from the state at a cost substantially below the price other companies charge. Nevertheless, the corporation believes that it is unconstitutional for the state to charge out-of-state purchasers more than resident purchasers. The out-of-state corporation, therefore, brings suit in federal court challenging the state pricing scheme. Assuming that the court has jurisdiction, should it uphold the constitutionality of the pricing scheme? A Yes, because as a market participant the state is free to charge nonresidents more than residents. B Yes, because the state is selling plastics to nonresidents at prices substantially below that of other companies. C No, because the scheme discriminates against nonresidents in violation of the Commerce Clause. D No, because charging nonresidents more for plastics than residents pay violates the Privileges and Immunities Clause guaranteeing benefits of state citizenship.

C

A developer owned a 240-acre parcel of land zoned for commercial and residential use. He prepared and recorded, after obtaining approval from all appropriate agencies, a subdivision plan that included a commercial center and a number of lots for single- and multi-family residences. The list of covenants, conditions, and restrictions recorded with the plan included provisions that required every building constructed in the subdivision to be of "simulated adobe style" architecture approved in advance by an association. A year later, the developer sold many of the lots in the commercial center, including several to a real estate firm. Each deed prepared by the developer contained a reference to the design restriction in the recorded plan. The developer also sold almost all of the residential lots, the deeds of which contained the same reference to the restriction. The following year, the real estate firm sold one of its lots to a burger franchise. The deed contained no reference to the design restriction. The franchise's prefabricated restaurant, complete with a giant burger logo mounted on the roof, was constructed over the weekend. A merchant, an original purchaser of one of the commercial lots, owned the lot next to the burger franchise. She did not learn of construction of the restaurant until she came in to work on Monday, and saw the giant burger logo. The merchant brings an action seeking a mandatory injunction compelling the burger franchise to demolish the restaurant. At trial, the merchant proves that the burger franchise did not seek or obtain approval of the association for its building. Should the court issue the injunction? A No, because destruction of the restaurant would be a tremendous waste of resources. B No, because the burger franchise's deed contained no restriction on the type of building that could be constructed on the lot. C Yes, because the restrictive covenant runs with the land. D Yes, unless the burger franchise can establish to the court's satisfaction that its restaurant design has at least as much aesthetic merit as any "simulated adobe style" design.

C (D) is incorrect because a court will not modify the covenant-it will enforce it or not enforce it, but will not substitute its judgment of what is aesthetically pleasing for the requirements of the covenant.

A smoothie retailer and a fruit processor entered into an oral agreement that provided that the processor would deliver to the retailer 100 barrels of fruit each month at a price of $10 per barrel, with delivery on the first of the month and payment of the $1,000 to a creditor of the fruit processor on the 15th of each month. However, when the agreement was reduced to a writing, the fruit processor's manager inadvertently wrote $20 per barrel, and neither party noticed before signing. The creditor first learned of the agreement between the parties when he received a copy of it the day after it was signed, showing that he was to receive $2,000 per month. One day later, the retailer discovered the mistake and alerted the food processor. The parties prepared a revised writing reflecting the correct contract price of $10 per barrel, and also agreed in writing that the retailer would receive a $2 per barrel discount the first month because it discovered the mistake by the fruit processor. The first delivery under the contract was made two days late, on the third of the month. On the 15th of the month, the creditor demanded payment of $2,000 from the retailer according to the terms of the original writing. If the retailer contends that it is not liable to pay the full $2,000, which of the following would NOT be relevant to its defense? A The parties had modified the contract to provide for a $2 per barrel discount the first month. B The parties had originally agreed that the price per barrel would be $10, and neither party noticed before signing that the manager of the fruit processor had inadvertently written $20 in the contract. C The fruit processor owed its creditor only $1,600. D The fruit processor was late with its first delivery.

C Any defense that the fruit processor might have with respect to the money it owed to the creditor would not provide the retailer with a defense. If the promisor has made an absolute promise to pay the third-party beneficiary (and not simply a promise to pay whatever the promisee owed him), the promisor cannot assert the promisee's defenses.

A seller entered a contract with a buyer under which the buyer agreed to purchase certain corporate securities for $500,000. The buyer refused to go through with the purchase, contending that the entire transaction was void because of federal securities statutes. The seller thus sold the securities to another party but was able to sell them for only $420,000. The seller filed an action against the buyer in federal district court, alleging that the transaction did not violate federal securities statutes and seeking $80,000 for the buyer's breach of contract. The seller and the buyer are both citizens of the same state. Does the federal court have subject matter jurisdiction over the action? A Yes, because the action raises the federal issue of whether the transaction violated federal securities statutes. B Yes, because the amount in controversy exceeds the required minimum for purposes of federal question jurisdiction. C No, because the buyer and seller are citizens of the same state and the action does not arise under federal law. D No, because the amount in controversy for purposes of federal question jurisdiction is not sufficient.

C Federal question jurisdiction is available when the plaintiff, in his well-pleaded complaint, alleges a claim that arises under federal law. Anticipation of a federal defense or the fact that federal law is implicated by the plaintiff's claim do not give rise to federal question jurisdiction; the plaintiff's claim must arise under federal law. Here, the plaintiff is anticipating that the defendant might raise federal securities laws as a defense, which is not enough to give rise to federal question jurisdiction.

A salvage company offered for sale Confederate dollars that had been recovered when the company recently raised a shipwreck off the coast of South Carolina. A purchasing agent for a private west coast museum purchased the bills, but he had represented that he was buying them for himself in hopes of obtaining a lower price. After purchasing the bills, the agent carefully packaged them and had them shipped to his museum. While the bills were in transit, the museum burned to the ground and its owner decided that she would not rebuild because most of her collections had been destroyed. When the bills arrived after the fire, the owner opened the package only to discover that the bills were too brittle for shipping by this method—three bills had disintegrated in transit. Undaunted, the owner took the remaining nine bills and had them mounted behind a glass frame so she could display them in her study. While the bills were being framed, the owner read on the Internet that a large cache of similar bills had just been discovered, and the market price for such bills had just been cut in half. Frustrated but still undaunted, the owner hung the framed bills in her study. Unfortunately, the salt water had reacted with the pigments in the bills in such a way that shortly after they had been exposed to indirect sunlight, all of the color in the bills faded almost completely away. No other Confederate bills raised from the ocean before had similar reactions; these bills appear to have been printed using substandard dyes. Which of the following facts would give the museum owner the best basis for rescinding the contract with the salvage company? A The bills were too brittle for transport. B The discovery of a large cache of similar bills a few days after the sale. C The bills' unusual reaction to indirect sunlight. DThe destruction of the museum before the bills arrived.

C Mutual Mistake

A man was tried in state court for possession of heroin. The prosecution offered in evidence five rolled-up toy balloons containing heroin, which police officers had found on a table in the man's apartment. At a hearing on the defense's motion to suppress, testimony was presented that established that the police had put the apartment under surveillance and had watched a police informant go to the door of the apartment, hand four balloons of heroin to the man, and leave. The police had then knocked on the apartment door, identified themselves as police officers, and demanded entrance. Having heard nothing for 30 seconds, the police had then broken down the door and entered the apartment, discovering the heroin. The police had intended to arrest the man for the purchase of heroin, a felony. When they had gotten inside the apartment, they discovered that the man had left by a back exit. He was later arrested at the nearby newsstand. The trial court denied the motion to suppress, and the case is on appeal following the man's conviction for possession of heroin. How should the appellate court rule? A Affirm the conviction on the ground that the error, if any, in admitting the heroin was harmless error. B Affirm the conviction on the ground that the police complied with the "knock and announce rule" even though no one was there to admit them. C Reverse the conviction on the ground that the man's Fourth Amendment rights (as applied to the states by the Fourteenth Amendment) have been violated. D Reverse the conviction on the ground that the "knock and announce rule" was not satisfied when the police announced their presence and identity to an empty residence.

C No exigent circumstances justified the warrantless arrest or the warrantless entry into the home. If the police had reason to believe the man was destroying evidence, they could have entered the home without a warrant to prevent the destruction under the exigent circumstances doctrine. But nothing in the facts gave the officers reason to believe that evidence was being destroyed. They knocked on the door, identified themselves, and demanded entrance. They heard no response or sounds of drugs being destroyed. Thus, there were no exigent circumstances. Evidence that is the fruit of an unlawful arrest may not be used against the defendant at trial because of the exclusionary rule.

A blacksmith ran a small forge in a tourist attraction depicting village life in the 1800s, and produced small metal trinkets for sale as souvenirs. A tourist came into the forge and started ridiculing the blacksmith, telling him that he was foolish for practicing such an out-of-date trade when modern equipment could produce the same trinkets faster and far more cheaply. Although he maintained a calm demeanor, the blacksmith was enraged by the time the customer finished and headed back out the door. He picked up an anvil and hurled it in the general direction of the customer. The anvil fell harmlessly to the ground after traveling maybe a foot. If the blacksmith is charged with assault, which of the following statements would be most helpful for his defense? Press Enter or Space to submit the answer A The blacksmith did not succeed in hitting the customer with the anvil, and he knew that it was impossible to do so. B The blacksmith knew that it was impossible to hit the customer with the anvil. C The customer did not see the blacksmith throw the anvil, and the blacksmith knew that it was impossible to hit the customer with the anvil. D The customer did not see the blacksmith throw the anvil.

C That the customer did not see the blacksmith throw the anvil, and that the blacksmith knew it was impossible to hit the customer with the anvil, would be most helpful to the blacksmith's defense. For purposes of the MBE, an assault is either (i) an attempt to commit a battery, or (ii) the intentional creation, other than by mere words, of a reasonable apprehension in the mind of the victim of imminent bodily harm. The fact that the blacksmith knew that it was impossible to hit the customer with the anvil negates the specific intent to commit a battery that is required for the first type of assault. (If the blacksmith knew that, when he threw the anvil, it was impossible to hit the customer, the blacksmith's conduct was not motivated by the intent to commit a battery against the customer.) The fact that the customer did not see the blacksmith throw the anvil negates the second type of assault because no apprehension of harm would have been created in the customer if he did not see the blacksmith throw the weight. Because the type of assault is not specified here, (C) is a better choice than (B) or (D) because both types of assaults are negated.

A landowner owns 15 acres of undeveloped property. He plans to build a stadium complex on the property to house a football team two years from now, but would like to open the 15 acres to public use for picnicking and similar activities until then. Which of the following would best accomplish the landowner's goal? A Dedicate the 15 acres for use as a public park. B Lease the 15 acres to the city for two years. C Grant the city an easement for public recreational uses for two years. D Covenant that the city may use the 15 acres for recreation for two years.

C The best way for the landowner to accomplish his goals is to grant the city an easement for recreational use for two years. An easement would allow the city to use the land only for the purposes provided for in the easement, and the landowner could limit the purposes to recreational uses. Thus, (C) is the best answer.

A buyer agreed to buy a limited edition guitar from a seller for $12,000 and a contract memorializing the agreement was signed by both parties. The next day, after the seller received an offer of $20,000 for the guitar, he called the buyer and said that he could not sell the guitar to him for $12,000. The buyer did not respond. On the delivery date, the buyer fails to tender $12,000 and the seller does not deliver the guitar. On these facts: A The seller can recover from the buyer for breach of contract. B The buyer can recover from the seller for breach of contract. C Neither the seller nor buyer can recover until one of the parties tenders performance. D The contract is terminated.

C The buyer can recover because the seller's phone call was an anticipatory repudiation. Anticipatory repudiation occurs where a promisor, prior to the performance time, unequivocally indicates that he cannot or will not timely perform, allowing the nonrepudiator the option of suspending performance and waiting to sue until the performance date, or to sue immediately. The seller's phone call was an unequivocal statement that he would not sell the guitar for $12,000. This repudiation excused the buyer's duty to tender $12,000 on the delivery date.

To combat rising unemployment, a state offered a $25,000 prize to anyone who could devise a scheme to create at least 200 jobs within the state and demonstrate its viability. While hiking through a national park within the state, a geologist noticed rock containing titanium. Knowing that titanium was commonly used in military aircraft built within the state and that mining and refining titanium could provide the state with thousands of jobs, the geologist chipped out a sample of the ore and took it back to the state employment division. After reviewing the geologist's ideas, the state announced in a press release that he was the first recipient of the $25,000 prize. Within a few days, the federal ranger in charge of the valley from which the sample was taken had the geologist arrested for violating a federal law making it illegal to remove any "plants, animals, or minerals from federal lands." The geologist was convicted and fined $5,000. He appeals the conviction to the federal court of appeals, claiming that the fine is unconstitutional. How should the court rule? A For the geologist, because the state has a compelling interest in reducing unemployment and the federal statute unreasonably interferes with the state interest. B For the geologist, because removing the ore was a purely intrastate act and had no effect on interstate commerce. C For the government, because the federal statute providing for the fine is constitutional under the Property Clause of Article IV, Section 3 of the federal Constitution. D For the government, because the federal statute providing for the fine is constitutional under the Commerce Clause.

C The court should affirm the geologist's conviction. The fine is constitutional under the Property Clause, which gives Congress the power to "make all needful rules and regulations respecting the territory or other property belonging to the United States." This power permits Congress to acquire and dispose of all kinds of property, and to protect its property with a law such as the one here.

The owner of a condominium hired a cleaning and junk removal service to clean his condominium after he moved. The parties agreed in writing that the company was to completely empty out the condominium, wash the walls and floors, and clean the appliances in exchange for $1,500. Shortly after beginning performance, the company assigned to a creditor its right to all monies due under the contract (i.e., $1,500), and the creditor promptly notified the condominium owner of the assignment. The condominium owner acknowledged the assignment. The company continued working, completely emptying out the condominium, washing the walls and floors, and cleaning all of the appliances except for the oven before quitting the job. It would cost $150 to hire a substitute to clean the oven. The condominium owner refuses to pay the creditor anything because of the cleaning service's breach. If the creditor sues the condominium owner, how much, if anything, is the creditor entitled to recover? A $1,500, the amount assigned, and the condominium owner may look to the company to recover for the minor breach. B The reasonable value of the labor and materials expended by the company on the portion of the job it did complete. C $1,350, on a theory of substantial performance. D Nothing, because the condominium owner's duty to pay is subject to a constructive condition precedent, and the assignee takes subject to the defense that the condition has not been satisfied.

C The creditor will be able to recover the contract price less damages for the company's minor breach. Generally, an assignee has whatever rights his assignor would have against the obligor. Similarly, the assignee is subject to any contract-related defenses that the obligor has against the assignor. Thus, the creditor will have whatever rights the company would have against the condominium owner. Here, the company completed all of the tasks that needed to be done except for one (i.e., cleaning the oven). If the work remaining on the contract is minor, the company will be seen as substantially performing its contract, and substantial performance will discharge its duty to perform and obligate payment by the condo owner. Because the facts state that the cost of finishing the job was relatively small (10% of the cost of the contract), it will probably be seen as a minor breach. Thus, the condominium owner cannot avoid payment of the contract price. However, despite the substantial performance, the other party to the contract may recover damages for the less than complete performance. Thus, the condominium owner will be able to offset his damages from the breach. The creditor then will be able to recover $1,350 (the contract price less the damages).

A grandfather told his granddaughter that she could have his house because he was moving to a retirement home, and entered into a valid contract to convey it to her. He promised her that he would have another wing added to the house in the back before turning it over to her, and entered into a written contract with a builder to construct the addition for his granddaughter. Before the grandfather had entered into the contract with the builder, the granddaughter had paid $5,000 for a 60-day option to purchase another house because she was not sure she would like the addition. However, when her grandfather showed her the plans for his house prepared by the builder, she liked it very much and decided to let her option to purchase the other house lapse. Shortly thereafter, the local zoning authority increased the minimum lot line setbacks, making it impracticable to put the addition on the back of the house. The builder offered to put an addition above the existing floor rather than in the back, and the grandfather agreed. After the granddaughter's option had lapsed, she discovered that the addition was now going up rather than in the back. She angrily demanded that the builder either build the addition according to the original specifications that she approved or pay her damages. The builder refused and the granddaughter filed suit. Who is more likely to prevail? A The granddaughter, because she was an intended beneficiary of the contract whose rights had vested. B The granddaughter, because the subsequent agreement between her grandfather and the builder to modify the construction was unsupported by consideration. C The builder, because he may raise all defenses that he had against the grandfather against the granddaughter. D The builder, because the granddaughter is merely an incidental beneficiary of the contract between the grandfather and the builder and, as such, has no power to enforce the contract against the builder.

C The granddaughter is an intended third-party beneficiary of the contract between the grandfather and the builder. (she was mentioned in the contract to build the home). Generally, a third-party beneficiary has rights under the contract as soon as she does something to vest her rights (manifests assent to the promise, brings suit to enforce the promise, or materially changes position by justifiably relying on the promise). Here, the granddaughter materially changed her position by justifiably allowing her option on the other house to lapse. Generally, once the third-party beneficiary's rights have vested, the original contracting parties may not modify the contract without the assent of the third-party beneficiary. However, the third-party beneficiary is subject to any defenses that the promisor could have used against the original promisee, and here the builder could have used the defense of impracticability against the promisee. Therefore, he could use that defense against the granddaughter to avoid having to pay damages for not building the house as he originally agreed.

A wholesale seller of mobile phones entered into a contract with a retailer, who was in the business of selling electronics, to sell 50 of a particular model phone at a cost of $200 apiece, or a total cost of $10,000. When the phones were delivered, the retailer discovered that 20% of them were defective. The retailer promptly sent the seller an electronic check for 80% of the contract price ($8,000). Did the retailer make a proper rejection? A Yes, because the phones were defective. B Yes, because the retailer paid for the phones that were not defective. C No, because the retailer accepted the phones and failed to seasonably notify the seller of any rejection due to defects. D No, because the retailer kept all of the phones and did not return the defective phones to the seller.

C The retailer did not properly reject, because he accepted the phones and failed to give proper notice of rejection. A buyer who receives nonconforming goods generally has the right to accept all, reject all, or accept any commercial units and reject the rest. Here, 20% of the phones shipped were defective, so the retailer had a right to reject. To properly reject, the rejecting party must, within a reasonable time after delivery and before acceptance, reject the goods or notify the seller of the rejection. Here, the retailer failed to reject the goods or notify the seller of the defects. He merely transferred payment for less than the contract price. Thus, the retailer cannot rely on the defect in claiming a breach, and his rejection was improper; i.e., he has accepted the goods.

A boy planned to rob the local currency exchange, but he needed a getaway car and somebody to be a lookout. He asked his sister, who immediately refused and warned him against carrying it out. The boy showed his sister a gun, threatening to shoot her if she did not go along with the plan. The sister consented to help, and the pair left for the currency exchange, with the sister driving. When they arrived, the sister left the car running so that she could get away and contact the police while her brother was inside, but the boy ordered her to go inside with him. During the robbery attempt, the clerk pulled out a gun, and the boy and the clerk shot and killed each other. If the sister is charged with the killing of the store clerk, should the jury find her criminally liable if it accepts the sister's version of the facts? A Yes, because duress is not a defense to murder. B Yes, because the clerk's death occurred in furtherance of and during the perpetration of a robbery in which the sister was participating. C No, because the sister lacked the requisite intent. D No, because the sister withdrew from the conspiracy to commit robbery and thus is not liable for any acts committed in furtherance of the conspiracy.

C The sister is not criminally liable for the store clerk's death. A defendant may, by virtue of her participation in a conspiracy, be liable for the crimes of all other conspirators if the crimes were committed in furtherance of the objectives of the conspiracy and were a natural and probable consequence of the conspiracy (i.e., were foreseeable). Furthermore, liability for felony murder would attach if the killing were committed during the course of a felony. Thus, the sister would be criminally liable for the killing of the clerk if she were a co-conspirator or if she were a principal to the underlying felony (the robbery). However, under the facts, the sister is neither because she lacked the requisite intent. Given the threat to her life, the sister did not freely enter into a conspiracy to commit robbery. (A) is incorrect because, while duress is not a defense to murder, it is a defense to the underlying felony, and thus is a defense to the felony murder charge. Here, the sister was acting under duress during the robbery, and thus she has a defense to the robbery charge.

A plaintiff brought an action in federal court against a state government, seeking monetary damages. The state moved to have the case dismissed for lack of jurisdiction, citing the Eleventh Amendment of the United States Constitution. Which of the following facts would support a denial of the state's motion? A The plaintiff is a private citizen of the defendant state. B The plaintiff is a Native American tribe. C The plaintiff is a neighboring state. D The plaintiff is a foreign government.

C The state's motion should be denied if the plaintiff is a neighboring state. The Eleventh Amendment does not bar actions by one state government against another state government.

A landowner devised her home "to my daughter for life, then to the eldest survivor of her two children, my grandson and granddaughter, for life, remainder to the eldest surviving offspring of the two grandchildren who is alive at the death of the last life tenant." After the landowner's death, the daughter lived in the family home for 15 years. Upon the daughter's death, both of her children were alive, so the home passed to the grandson, the eldest. He lived in the house for three years, and then conveyed it to the city historical society, which converted it into its headquarters and museum. Eight years later, the grandson died. At the time of his death, he was survived by his widow, his two sons, the granddaughter, and the granddaughter's daughter, who was the eldest of the niece and nephews. Four years after the grandson's death, the granddaughter's daughter brought an action for ejectment and to quiet title against the city historical society. The jurisdiction has a statutory period of adverse possession of 10 years, or five years if entry was made by the adverse possessor under color of title. How should the court rule in the granddaughter's daughter's action? A For the society, because it has occupied the home for the statutory period required for adverse possession. B For the society, because it purchased the home in fee simple absolute from the grandson. C For the granddaughter's daughter, because the society has not been in adverse possession for the requisite period. D For the granddaughter's daughter, because a purchaser of property from a life tenant cannot acquire a fee simple absolute through adverse possession.

C The statute of limitations that determines the time period for adverse possession does not run against the holder of a future interest (e.g., a remainder) until that interest becomes possessory, because the holder of the future interest has no right to possession (and thus no cause of action against a wrongful possessor) until the prior present estate terminates. Here, the society has possessed the home for eight years; however, as against the granddaughter's daughter, the holder of the remainder, the statute did not begin to run until the death of the grandson. Prior to the termination of the grandson's life estate, the granddaughter's daughter had no cause of action against the society because she had no right to possession. Upon the grandson's death, when the granddaughter's daughter's interest became possessory, the statute began to run against her. Thus, as against her, the society has not been in adverse possession for the requisite period.

Shortly after a professor at a state university completed her second year of teaching, she was informed that her contract was not being renewed for the following year. By state law, a professor does not acquire tenure until after she has completed three consecutive years of teaching. Before acquiring tenure, state law does not require either a statement of reasons or a hearing when a professor's contract is not renewed, and the university administration refused to give either to the professor. Which of the following, if established, sets forth the strongest constitutional argument that the professor could make to compel the university to furnish her a statement of reasons for the failure to rehire her and an opportunity for the hearing? A She purchased a home in anticipation of renewal of her contract, because most professors who had taught two years were rehired. B She had been voted the most popular professor on campus in each of her first two years of teaching. C She was the only teacher at the university whose contract was not renewed that year. D There is evidence to indicate that the decision not to rehire the professor was not based on her ability to teach.

C The strongest argument the professor could make is that the decision was not based on her ability to teach. The professor is an at-will employee, and under most circumstances may be discharged "for any reason or no reason at all." Thus, normally, evidence regarding the motives for dismissal is irrelevant. The question here, however, is what the strongest argument is that the professor could make, and (D) creates at least an inference that an impermissible motive might be present (gender, free speech, etc.).

A statute in the jurisdiction, which was enacted with the express purpose of preventing public employees from taking advantage of the status of undocumented immigrants, made it a felony to accept money or other benefits in exchange for issuing a state identification card. During an undercover investigation, an undocumented immigrant was recorded offering $500 to a clerk in exchange for issuance of a card. The clerk agreed to the deal and later that day exchanged the card for the money, after which both parties were arrested. In a jurisdiction following the common law approach to conspiracy, which of the following statements is correct? A. The clerk can be convicted of violating the statute and conspiracy to violate the statute, and the undocumented immigrant can be convicted of no crime. B The clerk can be convicted only of violating the statute, and the undocumented immigrant can be convicted of no crime. C The clerk can be convicted only of violating the statute, and the undocumented immigrant can be convicted as an accomplice to violation of the statute. D The clerk can be convicted of violating the statute and conspiracy to violate the statute, and the undocumented immigrant can be convicted of conspiracy to violate the statute.

C The undocumented immigrant cannot be convicted of a crime under the statute because it was enacted for his protection, and the clerk cannot be convicted of conspiracy under the statute because the undocumented immigrant, who would otherwise be liable as an accomplice, is not subject to conviction because of a legislative intent to exempt him. If a statute is intended to protect members of a limited class from exploitation or overbearing, members of that class are presumed to have been intended to be immune from liability, even if they participate in the crime in a manner that would otherwise make them liable. Thus, the undocumented immigrant would not be liable as an accomplice under the statute

A homeowner, a citizen of State A, hired an electrician, a citizen of State B, to fix the wiring in her basement and hired a gas worker, also a citizen of State B, to install a new gas stove in her kitchen. Unfortunately, the home caught fire and burned down while they were both working on their separate jobs. The homeowner sued the gas worker for negligence in federal court in State A, seeking $100,000. The homeowner promptly served the gas worker, and the gas worker timely filed an answer with the court. One month after filing the answer, the gas worker moved to file and serve a third-party complaint against the electrician, alleging that the electrician was the sole cause of the accident. Which of the following arguments is most likely to achieve the electrician's goal of dismissal of the third-party complaint? A The gas worker's motion for leave to file a third-party complaint is untimely and thus should be denied as a matter of law. B The court does not have subject matter jurisdiction over the third-party complaint because the electrician's claim and the gas worker's claim do not arise from a common nucleus of operative fact. C The gas worker's claim against the electrician is not a proper third-party claim. D Dismissing the gas worker's claim will not impede his ability to protect his rights in a separate action.

C Under Rule 14, a defendant may assert a third-party claim against "a nonparty who is or may be liable to it for all or part of the claim against it." In other words, a third-party claim must be a derivative claim; the third-party plaintiff must be seeking indemnification or contribution from the third-party defendant. Here, the gas worker's claim is not that the electrician must indemnify him or that the electrician is a joint tortfeasor who may be jointly liable under principles of contribution. Rather, the gas worker is alleging that he (the gas worker) is not liable and that the electrician is. Because the claim is not derivative, it is not properly asserted as a third-party claim under Rule 14.

A proud grandfather who planned to take pictures of his grandson's graduation purchased a camera from a camera store. He used the camera on several occasions over the next few weeks without incident, but when he used it on the day before his grandson's graduation, it caught fire and exploded, burning him and destroying an expensive coat he was wearing. Although the grandfather was in a great deal of pain because of his injuries, he insisted on attending his grandson's graduation. However, because he no longer had a workable camera, the grandfather hired a professional photographer to take pictures of the special day. In a breach of warranty action, which of the following represents the most that the grandfather may recover? Incorrect A The difference between the value of the camera accepted and its value if it had been as warranted. B The difference between the value of the camera accepted and its value if it had been as warranted, plus medical costs for treating the grandfather's burns. C The difference between the value of the camera accepted and its value if it had been as warranted, medical costs for treating the grandfather's burns, and the cost to replace the grandfather's coat. D The difference between the value of the camera accepted and its value if it had been as warranted, medical costs for treating the grandfather's burns, the cost to replace the grandfather's coat, and the cost of hiring the professional photographer.

C When a buyer accepts goods that turn out to be defective, he may recover as damages any "loss resulting in the normal course of events from the breach," which includes the difference between the value of the goods accepted and the value they would have had if they had been as warranted, plus incidental and consequential damages. Incidental damages resulting from the seller's breach include expenses reasonably incurred in inspection, receipt, and transportation, care, and custody of goods rightfully rejected. In this case, the grandfather incurred no incidental damages. Consequential damages resulting from the seller's breach include any loss resulting from general or particular requirements and needs of which the seller at the time of contracting had reason to know and which could not reasonably be prevented by cover or otherwise, and injury to person or property proximately resulting from any breach of warranty.

A building contractor entered into a contract with the local college to remodel a residence hall during the summer. As specified by the contract, the work had to be completed before the fall semester began at the beginning of September. Because the contractor received a great deal of other maintenance business from the college, his price of $400,000 was significantly lower than other contractors and he was not going to demand payment until the work was completed. By the end of the first week in August, the contractor had completed 75% of the project and had expended $350,000 in labor and materials. At that time, however, a labor dispute between the contractor and his employees prompted most of the workers to walk off the job. Because prospects for a quick settlement of the dispute were doubtful, the contractor informed the college that he would not be able to meet the completion deadline. A week later, the college obtained another contractor who was able to finish the project by the end of August. The college paid him $150,000, which included a substantial amount of overtime for his workers. The increase in value of the residence hall due to the remodeling was $425,000. The original contractor, who had not been paid, files suit against the college, which files a counterclaim against him. What should the contractor recover from the college? A Nothing, because the contractor breached the contract. B $200,000 in restitutionary damages, which is the difference between its expenditures and the amount the college paid the other contractor to complete the work. C $250,000 in restitutionary damages, which is the contract price minus the amount the college paid the other contractor to complete the work. D $275,000 in restitutionary damages, which is the difference between the value of the completed remodeling and the amount the college paid the other contractor to complete the work.

C Where a builder in a construction contract breaches during the construction, the nonbreaching party is entitled to the cost of completion plus compensation for any damages caused by the delay in completing the building. Most courts, however, will allow the builder to offset or recover for work performed to date to avoid the unjust enrichment of the owner. This restitutionary recovery is usually based on the benefit received by the unjustly enriched party. If substitute performance is readily obtainable, damages are measured by the unpaid contract price minus the cost of completion (up to the value of the benefit received by the defendant). Here, the contractor's duty to complete the project was not discharged by impossibility; he could have hired another contractor to take his place or yielded to his employees' demands. Hence, the contractor's failure to complete the remodeling constituted a breach of contract and resulted in the college having to expend $150,000 to have the building completed on time. However, the contractor did not receive any payments for the work that he did before breaching; the college would be unjustly enriched if it does not have to pay for any of this work. The benefit of the completed remodeling is measured by the contract price, $400,000, because a restitutionary recovery here would be based on the failed contract between the parties and substitute performance is readily obtainable. This amount is reduced by the $150,000 cost of completion that the college can recover from the contractor, leaving a net recovery of $250,000 for him.

The plaintiff was severely injured when a tire on her car blew out and caused her to lose control of the car. She filed an action in federal district court against the manufacturer of the tire, alleging that the tire was defective. The plaintiff intends to introduce remnants of the tire into evidence at trial as part of her proof that the tire was defective. In what way are the remnants subject to discovery? A Discovery is not available for such physical items, only for documents and electronically stored information. B The plaintiff does not have to disclose the existence or a description of the tire remnants unless the manufacturer asks about them in discovery requests. C Even without a request from the manufacturer, the plaintiff must provide the manufacturer with a description of the tire remnants. D The court will order the plaintiff to make the tire remnants available for the manufacturer to inspect only if the manufacturer shows good cause.

C Without waiting for a discovery request, a party must provide to the other parties copies or descriptions of tangible things that are in the disclosing party's possession or control and that the disclosing party may use to support its claims or defenses as an initial disclosure under Rule 26(a)(1). (Separately, the tire remnants would have to be disclosed under Rule 26(a)(3) as a pretrial disclosure of an item that is expected to be offered into evidence.)

A zoologist was moving to State A from State B, where she had lived her entire life. She was driving a truck and trailer that contained all of her possessions across the country when she was involved in a severe accident in State C. After being released from the hospital, but before suit was filed, the zoologist proceeded to State A, where she rented an apartment and found a job. The driver of an automobile involved in the severe accident intends to file a negligence action against the zoologist in federal district court. The driver is a citizen and resident of State D. In which federal district(s) is venue proper? A The District of State C only. B The District of State C and the District of State B. C The District of State C and the District of State A only. D The District of State C, the District of State D, and the District of State A.

C the District of State A is a proper venue because that is where the zoologist/defendant was domiciled at the time the case was filed

A company that was the leading supplier of home water filtration systems had a network of sales promoters who were under contract for two- or three-year terms and were compensated solely by commissions earned from sales and by occasional bonuses. Veteran promoters also earned commissions by recruiting other promoters for the company. One of the company's veteran promoters was contacted by a former top sales representative for another manufacturer who was looking for similar sales opportunities in the region. The sales rep knew that the promoter might be able to get her a position with his company, which was looking for additional promoters. At the time he met with the sales rep, the promoter's contract with the company had one more month to run. When the promoter's contract with the company expired, he announced that he was forming his own business to market a different line of water filtration systems manufactured by a competitor of the company, and that the sales rep would be in charge of his promotional network. The company brought an action against the promoter for interference with business relations for hiring the sales rep. At a preliminary hearing, the parties stipulated to the above facts and that the promoter was an independent contractor rather than an employee of the company. The promoter then filed a motion for a summary judgment in his favor. Should the court grant the promoter's motion? A Yes, because the sales rep had no business relationship with the company at the time the promoter's alleged interference occurred. B Yes, because the promoter was an independent contractor rather than an employee of the company. C No, because the jury could find that the means the promoter used to obtain the sales rep were not privileged. Incorrect D No, because the jury could find that the promoter breached his contract with the company by meeting with the sales rep.

C the court should not grant the promoter's motion because the jury could find that the promoter used improper means, while working for the company, to divert the sales rep for his own purposes. To establish a prima facie case for interference with business relations, the following elements must be proved: (i) existence of a valid contractual relationship between plaintiff and a third party or a valid business expectancy of plaintiff; (ii) defendant's knowledge of the relationship or expectancy; (iii) intentional interference by defendant that induces a breach or termination of the relationship or expectancy; and (iv) damage to plaintiff. Thus, a plaintiff has a cause of action for interference with probable future business relationships for which the plaintiff has a reasonable expectation of financial benefit. On the other hand, an interferer's conduct may be privileged where it is a proper attempt to obtain business for the interferer, particularly if the interference is only with a prospective business relationship rather than with an existing contract. What is proper depends on various factors, including the means of persuasion used. Here, the promoter's conduct would not be privileged if the jury were to find that he improperly used his position with the company to develop a relationship with the sales rep.

A bar prep company discovered that its copyrighted content was being used in an online simulated exam that its competitor was administering in a few days. The company filed a petition for an ex parte order to direct the competitor to immediately remove that content from its website. The company submitted an affidavit specifying why immediate and irreparable injury will result if the exam is administered, and offered to provide security for any costs or damages incurred by the competitor if it was determined that the order was wrongfully issued. Should the court issue the order? A Yes, because the company submitted an affidavit with specific facts showing immediate and irreparable injury. B Yes, because the competitor will receive notice of the order once it is issued by the court. C No, because a court cannot issue an injunction unless the adverse party has notice of the hearing. D No, because the company has not provided sufficient certification for obtaining an ex parte order.

D

A company operated a small amusement park on property it owned near a residential neighborhood. On a day when the park was closed, a 10-year-old girl snuck into the park with some friends by climbing over a chain link fence. While climbing on one of the carnival rides, the girl slipped and cut her leg on an exposed gear assembly, sustaining serious injuries. Through her guardian ad litem, the girl brought suit against the company to recover damages for her injuries. At trial, she presented evidence of the accident and her injuries. In defense, the company established that the girl read and understood the "No Trespassing" signs that were attached to the fence. The company also established that it had not had any previous reports of children sneaking into the park when it was closed. Before submission of the case to the jury, the company moved for summary judgment. Is the court likely to grant the company's motion? A Yes, because the girl was a trespasser who the company had no reason to anticipate would be on the property. B Yes, because the girl knew she was trespassing and was old enough to recognize the danger. CNo, because the jury could find that the company should have foreseen that children would sneak into the park. D No, because the appeal of the carnival rides attracted the girl into the park.

D

A hotelier entered into a contract with a merchant to have the merchant build and deliver custom furniture having a total cost of $1 million. The hotelier is a resident of State A and the merchant is a resident of State B. The merchant completed the furniture but the hotelier refused delivery, claiming that the furniture was not as the hotelier and merchant had agreed. The hotelier then filed an action against the merchant in federal district court, claiming breach of contract and seeking the return of a $100,000 deposit. The merchant filed an answer, denying liability and denying that it breached the contract. At the same time, the merchant filed and served a counterclaim asserting that the hotelier had breached the contract and seeking the $900,000 balance due. A week after being served with the merchant's answer and counterclaim, the hotelier filed a voluntary dismissal of the action. Is the hotelier's dismissal effective? A Yes, because the party that commenced an action may always terminate it. B Yes, because a plaintiff may voluntarily dismiss an action of right if the plaintiff does so within 21 days of the defendant filing an answer. C No, because, while the hotelier otherwise would have had the right to dismiss the action, the hotelier cannot voluntarily dismiss the action because of the merchant's pending counterclaim. D No, because, after a defendant files an answer, an action may be dismissed only by a stipulation signed by all parties or by an order of the court.

D

A pedestrian from State A filed a civil tort action against a driver from State B in a State A state court, alleging that the driver struck the pedestrian when thepedestrian was crossing the road in State A. The pedestrian alleges damages of $100,000 in accordance with state procedure. State A and State B each have one federal court. Which of the following best explains the procedure the driver should follow to remove the case to federal court? A The driver should file a motion to transfer venue in the state court where the action was filed. B The driver should file a motion to transfer venue in the federal court for State A. C The driver should file a notice of removal in the state court where the action was filed, and then serve a copy on the driver. D The driver should file a notice of removal in the federal court for State A, and serve a copy of the notice on the driver and file a copy with the state court where the action was filed.

D

An antique lover spotted a beautiful Early American bedroom ensemble at her favorite antique store. The ensemble included a bed, a mirror, and two dressers. Over a period of several weeks, the shop owner and the antique lover negotiated over a price, but they were unable to come to an agreement. On April 3, the shop owner and the antique lover signed a statement whereby the shop owner offered to sell to the antique lover an Early American bedroom ensemble, recorded as items 20465, 20466, 20467, and 20468 in the shop's registry, if the parties agree upon a price on or before April 12. On April 6, the shop owner sent a letter to the antique lover, telling her that she could have the bedroom ensemble for $22,000. Also on April 6, the antique lover sent a letter to the shop owner telling him that she was willing to pay him $22,000 for the bedroom ensemble. Both parties received their letters on April 7. Without assuming any additional facts, which of the following statements is most correct as of April 8? A The shop owner and the antique lover had a valid contract from the moment the letters of April 6 were mailed. B A contract exists between the shop owner and the antique lover, because the shop owner, a merchant, sent the antique lover an offer in writing. Incorrect C A contract exists between the shop owner and the antique lover, because the crossing offers were identical and received before April 12. D No contract exists between the shop owner and the antique lover, because of a lack of mutual assent.

D

An environmental group, wishing to stop the issuance by a federal agency of a mining permit to a coal company, commences an action in federal court against the federal agency, seeking, among other things, a permanent injunction barring the issuance of the permit to the coal company. If the coal company seeks to join the litigation as a matter of right, must the federal court grant the motion? A No, because intervention of outside parties is a matter within the sole discretion of the judge. B No, unless the coal company has been given an unconditional right to intervene by a federal statute. C Yes, because the coal company has an interest in getting the mining permit. D Yes, unless the court concludes that the coal company's interest in getting the permit is adequately protected by the federal agency.

D

An office furniture supplier filed a breach of contract action against a law firm in federal district court to recover the balance due on an account for furniture it supplied. The law firm filed a motion to dismiss the action on the ground that service of process was improper. Following a hearing, the court held that service was proper and sufficient. Two more months passed without the law firm filing an answer. The supplier then filed a motion to have the clerk of court make an entry of default, and the clerk did so. What procedure must the supplier follow to obtain a default judgment against the law firm? A File a motion to have the clerk of court enter the default judgment, and the clerk may do so without the law firm receiving any further notice of the motion. B File a motion to have the clerk of court enter the default judgment, and the clerk may do so as long as the law firm receives additional notice of the motion for default judgment. C File a motion to have the judge enter the default judgment, and the judge may do so without the law firm receiving any further notice of the motion for default judgment. D File a motion to have the judge enter the default judgment, and the judge may do so as long as the law firm receives additional notice of the motion for default judgment.

D

Congress enacted a statute requiring state-supported institutions of higher education that provide federal student loan funds to their students to fund women's sports according to a complex formula intended to fairly support women's athletics and remedy past funding discrimination. Under the formula, a particular state military school will be required to allocate 25% of its athletic budget to its female athletics programs even though only 10% of the school population is female. A male student whose athletic program will be discontinued because of the budget allocation filed suit in federal court challenging the federal statute on various constitutional grounds. Is the court likely to find that the statute is constitutional? A No, because the government will be unable to prove that the discriminatory funding requirements of the statute are necessary to achieve a compelling government interest. B No, because the federal government does not have the power to dictate the budget allocations of state-supported educational institutions. C Yes, because remedying past discrimination is a legitimate government interest, and the student will be unable to prove that the statute's funding requirements are not rationally related to that interest. D Yes, because the government will be able to prove that the statute's funding requirements are substantially related to an important government interest.

D

The defendant and an accomplice were on trial together for burglary. Both had given confessions implicating themselves and their accomplice. At trial, the defendant maintained that his confession had been obtained through improper coercion by the police. For the purpose of countering the claim of coercion, the prosecution seeks to place the accomplice's confession into evidence. After objection by the defendant's counsel, the judge agrees to issue a limiting instruction to the jury that the confession is to be considered only with regard to the question of whether the defendant's confession was coerced. May the accomplice's confession be admitted under that condition? A No, because admission of the confession violates the defendant's right of confrontation. B No, unless the accomplice takes the stand and subjects himself to cross-examination regarding the confession. C Yes, as long as all portions of the confession referring to the defendant can be eliminated. D Yes, because the judge's instruction limits consideration of the confession only to the issue of coercion.

D

A man went to his local sporting goods store and told the salesperson that he wanted a tennis racket that was very high-end. The salesperson showed him a racket that he said was made of the finest titanium and would probably last for years. The man bought the racket and left the store. After playing with the racket for three days, it suddenly snapped in two when he hit a hard shot. He showed it to the tennis pro at his club, who informed him that the racket was painted plastic. If the man sues the sporting goods store, which of the following best describes the legal basis for his suit? A The store breached the implied warranty of fitness for a particular purpose and an express warranty that the racket was made of titanium. B The store breached the implied warranty of merchantability and an express warranty that the racket would last for years. C The store breached both the implied warranty of fitness for a particular purpose and the implied warranty of merchantability. D The store breached the implied warranty of merchantability and an express warranty that the racket was made of titanium.

D An express warranty will arise from any statement of fact or promise. Here, the salesperson said that the racket was made of titanium. This is a statement of fact that will give rise to a warranty. An implied warranty of merchantability will arise in every sale by a merchant unless disclaimed. To be merchantable, goods must be fit for ordinary purposes, and arguably a racket that breaks right away because it is made of plastic is not fit for ordinary purposes.

A driver was killed in a two-car accident in State A. The deceased driver was a citizen of State B, and the driver of the other car is a citizen of State A. The executor of the deceased driver's estate filed a negligence action in federal district court on behalf of the deceased driver against the State A driver, seeking $250,000. The executor is a citizen of State A. Does the federal court have diversity of citizenship jurisdiction over this action? A No, because the executor and the defendant are both citizens of State A. B No, because federal courts will not exercise diversity of citizenship jurisdiction over probate matters. C Yes, because the executor of the plaintiff's estate is a citizen of State A and the deceased plaintiff is a citizen of State B. D Yes, because the deceased plaintiff was a citizen of State B and the defendant is a citizen of State A.

D For diversity purposes, a legal representative of a decedent assumes the state citizenship of the decedent.

A landlord brought suit against a tenant in federal court for overdue rent payments on a commercial lease. The landlord sought to recover on the six rent installments that were past due and unpaid at the time of the suit. The landlord won the case, and judgment was entered in her favor. The lease has an acceleration clause that states that all future rent payments become due if the tenant falls behind three months or more. The landlord now files suit against the tenant for the remaining rent payments. The tenant moves to dismiss, asserting that the landlord's claim is barred by claim preclusion (res judicata) principles. Should the tenant's motion to dismiss be granted? A No, because a landlord may choose when to sue on an acceleration clause. B No, because the two suits do not involve the same cause of action. C Yes, because a landlord may not sue on later installments of an installment contract. D Yes, because the two suits arose out of the same transaction or occurrence.

D Generally, a claimant is required to assert all causes of action arising out of the same transaction or occurrence that is the subject matter of the claim. In the situation of installment payments, the claimant is required to sue on all installments due at the time of the suit. If there is an acceleration clause, the claimant must sue for all installments.

A manufacturer of down coats and jackets entered into a written agreement with a distributor, whereby the distributor agreed to distribute the manufacturer's products statewide for a one-year period to begin on June 1. Before the manufacturer signed the distribution contract with the distributor, the distributor told the manufacturer that their deal was exclusive, but nothing to that effect was in the written agreement. However, in the outerwear industry it has been a custom for many years for distributors to distribute only one brand of outerwear. On September 1, the distributor began distributing coats and jackets manufactured by one of the manufacturer's chief competitors. These coats and jackets were sewn with man-made fabrics, were as warm as the manufacturer's jackets, and were less bulky. The competitor's advertising campaign throughout the state emphasizes that "you don't have to look fat to stay warm." Seasonally adjusted sales figures showed that the manufacturer's sales in the state dropped 6% after its competitor's products were introduced. The manufacturer of the down coats and jackets complained to the distributor, demanding that it stop distributing the man-made coats and jackets made by the manufacturer's competitor. The distributor refused, and the manufacturer of the down coats and jackets brought suit against the distributor. Which of the following facts would provide a basis for the manufacturer's best case against the distributor? A The competitor's advertising campaign throughout the state alluding to the unattractive bulkiness of the manufacturer's coats and jackets. B The 6% drop in seasonally adjusted sales figures in the state after the competitor's products were introduced. C The distributor's oral statement to the manufacturer about their deal being exclusive. D The long-standing custom in the outerwear industry for distributors to distribute only one brand of outerwear.

D Of all the alternatives listed, (D) is the only one that presents any real basis for supporting the manufacturer's case. One of the general rules of contract construction, including contracts for goods under the UCC, is that courts will look to see what custom and usage are in the particular business and in the particular locale where the contract is either made or to be performed. The manufacturer could claim that when he and the distributor entered into the distribution contract, both parties implicitly understood that the custom of distributing only one brand of outerwear would be followed in their transaction. Under such circumstances, the manufacturer may be able to successfully assert that the distributor's distribution of the competitor's outerwear constitutes a breach of contract. The fact that the competitor's advertising campaign at least impliedly denigrates the appearance of the manufacturer's outerwear (as in (A)), or that the manufacturer's sales have dropped since the introduction of the competitor's products (as in (B)), establishes no cause of action against the distributor. Absent some provision in the contract, or some reference to custom and usage as mentioned in (D), there is no basis for holding that the distributor was prohibited from distributing other companies' products, or that the distributor can be held liable for a decline in the manufacturer's sales figures due to sales or advertisements made by a company whose products are being distributed by the distributor. Under UCC section 2-202, a party cannot offer consistent additional terms if the writing was intended as a complete statement of the terms of the agreement. (In contrast, evidence of custom in the trade can be offered regardless of the completeness of the written agreement.)

A large wholesale dealer in produce had never done business with a certain greengrocer who operated a small chain of markets in the Midwest. They entered into a written agreement whereby the wholesale dealer agreed to supply to the greengrocer the "fuzzy" variety of peaches at $35 per 50-pound lot. The agreement contained a provision stating that the greengrocer will buy "as many 50-pound lots of fuzzy peaches as the greengrocer chooses to order." Assuming that the greengrocer has not yet placed any orders for peaches with the wholesale dealer, is this agreement between the parties enforceable? A Yes, because it is a valid requirements contract and, as such, is enforceable under the Uniform Commercial Code. B Yes, because the Uniform Commercial Code will imply reasonable terms. C No, because the total quantity of the contract is not specified. D No, because there is no manifestation of commitment on the greengrocer's part.

D The agreement is not enforceable because the greengrocer's promise is illusory. For a contract to be enforceable, consideration must exist on both sides, i.e., each party's promise must create a binding obligation. If one party has become bound but the other has not, the agreement lacks mutuality because one of the promises is illusory. Here, the wholesale dealer has promised to supply the greengrocer with fuzzy peaches at a fixed price. The greengrocer, however, has not promised to order any peaches from the wholesale dealer. Even if the greengrocer decides to sell fuzzy peaches, it has not bound itself to order them from this particular wholesale dealer. The illusory nature of the greengrocer's promise makes the agreement unenforceable on consideration grounds.

A state statute makes criminal "all speechmaking, picketing, or public gathering of any sort on the steps of the supreme courthouse Monday through Friday, between the hours of 8:30 a.m. and 4:30 p.m., when court is in session." A citizen is upset about a supreme court decision that was just released and stands on the steps of the courthouse at noon, while court is in session, handing out leaflets and exhorting passersby to vote the state supreme court justices out of office. If the citizen is prosecuted for violation of the statute, which of the following best describes the applicable burden of proof? A The state will have to show that there was a compelling need for the statute and that no less restrictive alternatives existed to meet that need. B The state will have to show that the statute was narrowly tailored to serve an important government interest and leaves open alternative channels of communication. C The citizen will have to show that there was no compelling need for the statute and that less restrictive alternatives were available to accomplish the same goals. D The citizen will have to show that there was no reasonable basis for enacting the statute.

D The citizen will have to show that the statute was not reasonably related to a legitimate government purpose. Other than streets, sidewalks, parks, and designated public forums, most public property (including a court building and its grounds) is considered to be a limited public forum or a nonpublic forum. The government can regulate speech in such a forum to reserve the forum for its intended use. Regulations will be upheld as long as they are (i) viewpoint neutral, and (ii) reasonably related to a legitimate government purpose. Here, the statute prohibited public gatherings on the steps of the courthouse at specified times while the court was in session, which appears to be a reasonable, viewpoint neutral effort to preserve government property for its intended use. The citizen would have the burden of proving that there was no reasonable basis for the statute.

A homeowner hired a contractor to make some improvements on his house. They entered into a written contract providing that the contractor would do the improvements for $5,000. Shortly after the contract was signed, the contractor told the homeowner to give the money to his (the contractor's) daughter when the job was finished, adding, "She is getting married soon and I want her to have a nice wedding present from me." The daughter was aware that her father made this statement to the homeowner. She married, but soon thereafter the contractor told the homeowner to pay him the $5,000, and not the daughter, because his son-in-law had a gambling problem and would probably use the money to bet at the racetrack. What is the best argument in favor of the daughter's being able to enforce a contract for $5,000 in her favor? A Statute of Frauds. B Parol evidence rule. C The daughter was an intended third-party beneficiary. D The daughter married in reliance on the promise.

D The daughter's best argument to enforce the contract in her favor is that she married in reliance on the contract (detrimental reliance), although she will probably be unsuccessful. Here, the daughter was a gratuitous assignee (because she gave no consideration) and her rights under the contract were revoked. Thus, her strongest argument will be one that nullifies the revocation. Under the doctrine of detrimental reliance, a promise will be enforced to the extent necessary to prevent injustice if it was made with a reasonable expectation that it would induce reliance, and such reliance was in fact induced. The problem with this argument here is that it is not clear that the daughter relied on the promise to give her $5,000, because she already had planned to get married. However, none of the other choices is a possible argument, so (D) is her best choice.

The defendant was arrested, given Miranda warnings, and charged with burglary. At the police station, he telephoned his mother and asked her to come to the station to post bail. Instead, his mother immediately called the family attorney. In the meantime, the police had begun questioning the defendant. Although he never told the police to stop the questioning, his answers were at first vague or clearly unresponsive. During the course of the questioning, the family attorney phoned the station and told the police that she had been hired to represent the defendant and would be there in half an hour. The police did not inform the defendant of the attorney's call. Ten minutes later, the defendant admitted to committing the burglary, and signed a statement to that effect prepared by the police. The attorney arrived a few minutes later and advised the defendant to remain silent, but he told her that he had already signed a confession. How should the court rule on the attorney's pretrial motion to exclude the confession as evidence at trial? A Grant the motion, because the police had a duty to inform the defendant that an attorney was coming to represent him. B Grant the motion, because the defendant has been deprived of his Sixth Amendment right to counsel. C Deny the motion, because the defendant's statement admitting the crime was voluntary. D Deny the motion, because the defendant waived his Miranda rights.

D The defendant's confession should be admitted because he waived his Fifth Amendment privilege against compelled self-incrimination after receiving Miranda warnings. Miranda v. Arizona requires that a person in custody be informed of his right to remain silent and his right to the presence of an attorney during questioning. A suspect may subsequently waive his rights by making a confession, as long as the waiver was knowing and voluntary.(C) is incorrect even though it is true that the defendant made a voluntary statement. Due process requires that for confessions to be admissible, they must be "voluntary," based on the totality of the circumstances, and here all of the circumstances indicate that the defendant's confession was voluntary. However, even a voluntary confession will be inadmissible if it was obtained in violation of Miranda rights. (D) is therefore a better choice than (C).

A state's highway speed limits were 65 miles per hour in its flat land regions and 55 miles per hour its mountainous regions. To reduce traffic fatalities and combat the fact that most of the vehicles on state highways were exceeding posted speed limits, the state legislature proposed banning the use of radar detectors. Citizens in the mountainous regions of the state, where most of the state's highway fatalities occurred, generally supported the ban, but citizens in the flat regions of the state opposed the ban, so the legislature adopted a law banning use of radar detectors on any road with a speed limit below 60 miles per hour. A driver whose car was equipped with a radar detector lived in the mountainous region of the state but frequently drove to the state's flat region. While on a mountain highway with a posted speed limit of 55 miles per hour, the driver was pulled over by a state trooper for speeding. While approaching the driver's car, the state trooper noticed that the driver's radar detector was turned on. The trooper ticketed the driver for both speeding and illegal use of a radar detector. The driver challenges his ticket for use of the radar detector, arguing that it is unfair to allow people in the flat lands to use radar detectors while prohibiting residents of the mountainous region from using them. Which of the following statements is correct regarding the burden of proof in such a case? A The state will have to prove that the ban serves a compelling state interest. B The state will have to prove that the ban is rationally related to a legitimate state interest. C The driver will have to prove that the ban does not serve a compelling interest. D The driver will have to prove that the ban is not rationally related to a legitimate state interest.

D The driver will have to prove that the ban is not rationally related to a legitimate state interest. Whether the law here is examined under the substantive provisions of the Due Process Clause or the Equal Protection Clause, the analysis is the same: If no fundamental right or suspect or quasi-suspect classification is involved, the law will be assessed under the rational basis standard. Under that standard, government action will be upheld unless a challenger can prove that it is not rationally related to a legitimate government interest. Here, the right involved (using a radar detector) is not fundamental, and no suspect or quasi-suspect classification is involved. Thus, the regulation will be assessed under the rational basis standard.

A defendant was on trial for burglary, and he took the stand in his own defense. On direct examination, the defendant vigorously denied having committed the burglary. Also on direct examination, the defendant stated that his last regular employment was as a bookkeeper for a corporation. On cross-examination, the prosecutor asked the defendant if he had embezzled funds from the corporation. The defendant denied that he had embezzled from the corporation or from anyone else. The prosecutor then wanted to call a police officer to the stand to testify that when she arrested the defendant for embezzlement, the defendant admitted to the officer that he had embezzled money from the corporation. Assuming that the defendant has not yet been tried on the embezzlement charges, may the prosecutor call the officer to the stand? A Yes, but only for purposes of impeachment. B Yes, both for impeachment of the defendant and as substantive evidence. C No, because the defendant has not yet been convicted of embezzlement. D No, because the evidence would be extrinsic.

D The officer may not testify about the embezzlement because it constitutes impeachment by extrinsic evidence of a specific instance of misconduct. A specific act of misconduct offered to attack the witness's character for truthfulness can be elicited only on cross-examination. If the witness denies the act, the cross-examiner cannot refute the answer by calling other witnesses or producing other evidence. Because the alleged embezzlement is admissible, if at all, only as impeachment evidence, when the defendant denied it the prosecutor could not call the officer to testify.

Congress passed a law allowing widespread oil exploration on federal lands in the western United States. A large deposit of oil sand was discovered in one western state and Congress authorized an oil sand refining plant to be built on federal park land within the state. The refinery was built in compliance with federal pollution regulations. Pursuant to state law, the plant manager allowed the state to inspect the plant before putting it into operation. Because state refinery standards were more strict than the federal standards (in order to better protect state citizens from pollution associated with refineries), the refinery did not pass the inspection, and the state inspector refused to give the manager a permit to run the refinery. The refinery manager nevertheless began to run the refinery and was fined by the state. Which of the following is the manager's best defense against imposition of the fine? A The state does not have a compelling interest in regulating the refinery, because it is within a federal park. B The state regulation is invalid because Congress has preempted the field of pollution control. C The state pollution regulation is invalid because it is inconsistent with the state's compelling interest in providing jobs. D The state law violates the principles of intergovernmental immunity as applied to the manager.

D The states have no power to regulate the activities of the federal government unless Congress consents to the regulation. Thus, instrumentalities and agents of the federal government are immune from state regulations that interfere with their federal functions. Here, the regulation clearly interferes with the manager's duties to run the refinery. While it might be argued that the manager agreed to comply with the state regulations, because he allowed the state inspection, nothing indicates that Congress consented, and so the state regulation cannot be applied to the manager.

A contractor filed a breach of contract action against a supplier in federal district court, seeking compensatory damages. The contractor does not seek, and under applicable law cannot recover, punitive damages. The supplier admits the existence of the contract but denies breach. The contractor served on the supplier an interrogatory asking the supplier to state his net worth. Assuming the supplier objects to the interrogatory and the contractor files a motion to compel an answer, will the court require the supplier to answer? A Yes, because the requested information is relevant. B No, unless the contractor seeks the information to determine whether the supplier has sufficient assets to make the action worthwhile. C No, because the request seeks privileged information. D No, because the requested information is not relevant to the claim or defense of any party.

D discovery may be had of any nonprivileged matter that is relevant to any party's claim or defense and proportional to the needs of the case

Legislation permitting states to completely ban the sale of cigarettes and other tobacco products was passed by Congress and upheld by the United States Supreme Court. Following the lead of other states, a state legislature declined to enact a complete ban on tobacco products. However, it passed a compromise measure that banned all advertisements for cigarettes and tobacco products by any print or broadcast media located in the state. A state tobacco distributor that wished to advertise in local newspapers brings an action in federal court to challenge the state statute. How is the court likely to rule? A The statute is unconstitutional, because by choosing not to exercise its right to impose a complete ban on the sale of tobacco products, the state can no longer claim that the regulation of advertising serves a substantial government interest. B The statute is unconstitutional, because the right to ban all truthful advertising for a product is not automatically justified by the right to ban the sale of a product entirely. C The statute is constitutional, because the state's power to ban advertising for a product is implicit in its power to ban the product altogether. D The statute is constitutional, because the ban on advertising constitutes a restriction on commercial speech that is rationally related to the legitimate state interest in reducing the use of tobacco products.

The court will probably find the statute unconstitutional as an improper restriction of commercial speech. If the speech regulated concerns a lawful activity and is not misleading or fraudulent, the regulation will be valid if it (i) serves a substantial government interest, (ii) directly advances the interest, and (iii) is narrowly tailored to serve the substantial interest. While this test does not require that the least restrictive means be used, there must be a reasonable fit between the legislation's end and the means chosen. The greater the restriction on speech, the less likely it will be deemed to be reasonable. A complete ban on truthful advertising of a lawful product is very unlikely to be upheld because such a restriction is not narrowly tailored. [See 44 Liquormart, Inc. v. Rhode Island (1996)] Hence, the complete ban on advertising of tobacco products probably will be an unconstitutional infringement on freedom of speech.


Set pelajaran terkait

Lifting and moving, Chap 3. EMT!

View Set

Chromosome Variation (Chapter 8)

View Set

Chapter 2: Communication and Personal Identity

View Set

You're welcome B*tches!! A Sociology of the Family Inquisitive

View Set

Chapter 17 Slides: Outlet Selection and Purchase

View Set

HESI Fundamentals Practice Exam 2023

View Set

10.2 COMMON WORKPIECE MATERIAL TYPES - BASIC MILL OPERATOR

View Set

Proximal and Distal Attachments of Foot, Ankle and Lower leg

View Set